60
Cornea/External Disease Question 1 of 130 With regard to attempting to achieve coagulation of the corneal vessels in lipid keratopathy, which of the following statements would be least likely. Treatment could trigger a corneal graft rejection in a postkeratoplasty patient. The endpoint is slight blanching. A 50-j.1ill to 200-j.1ill spot size should be used. Efferent vessels (venules) should be treated first. Please select an answer Feedback: Corneal neovascularization should only be treated if it will improve the patient's vision or visual outcome. For example, deep vessels are associated with graft rejection, so occlusion of such vessels prior to penetrating keratoplasty may be beneficial. Prior to considering laser therapy, any stimuli for neovascularization should be eliminated if possible, or the vessels are very likely to return. If there is active inflammation of the cornea, it should be controlled. For best results, the afferent (arteriole) vessels should be treated first. This will induce less stasis of blood and allow better treatment of all vessels. All vessels should be treated. Often more than one session is needed to treat all vessels. Topical anesthesia, pilocarpine (to reduce the chance for damage of the lens or retina), and a laser contact lens are placed on the eye. Laser settings include a 50-pm to 200-pm spot size, depending on the caliber of the blood vessel, and a (Ll-second duration. Power is set at 400 mW and increased as necessary to look for blanching of the vessels for about 2 mm to 3 mm. Potential complications with this therapy include intracorneal hemorrhaging, iris atrophy, corneal thinning, recurrence of neovascularization, and corneal graft rejection. Question 2 of 130 According to Eye Bank Association of America medical standards, which eye could not be used for corneal transplantation? Eyes from a 75-year-old donor with 2200 cells/mm An eye from a donor with diabetic retinopathy An eye that has undergone phacoemulsification and has a normal endothelial cell count An eye that has undergone excimer laser photorefractive keratectomy (PRK) Please select an answer Feedback: Current Eye Bank Association of America standards allow for the use of tissue from older donors and eyes that have had previous intraocular surgery as long as the tissue rating itself is good. Diabetes may affect the corneal endothelium; hence it is a relative but not absolute contraindication for transplantation. Any corneal refractive surgery is a contraindication for use in corneal transplantation. The surgeon carries the ultimate responsibility for determining the suitability of transplant tissue for use in a particular recipient.

Cornea/External Disease

  • Upload
    others

  • View
    1

  • Download
    0

Embed Size (px)

Citation preview

Page 1: Cornea/External Disease

Cornea/External Disease

Question 1 of 130 With regard to attempting to achieve coagulation of the corneal vessels in lipid keratopathy, which of the following statements would be least likely.

Treatment could trigger a corneal graft rejection in a postkeratoplasty patient.

The endpoint is slight blanching.

A 50-j.1ill to 200-j.1ill spot size should be used.

Efferent vessels (venules) should be treated first.

Please select an answer Feedback: Corneal neovascularization should only be treated if it will improve the patient's vision or visual outcome. For example, deep vessels are associated with graft rejection, so occlusion of such vessels prior to penetrating keratoplasty may be beneficial. Prior to considering laser therapy, any stimuli for neovascularization should be eliminated if possible, or the vessels are very likely to return. If there is active inflammation of the cornea, it should be controlled. For best results, the afferent (arteriole) vessels should be treated first. This will induce less stasis of blood and allow better treatment of all vessels. All vessels should be treated. Often more than one session is needed to treat all vessels. Topical anesthesia, pilocarpine (to reduce the chance for damage of the lens or retina), and a laser contact lens are placed on the eye. Laser settings include a 50-pm to 200-pm spot size, depending on the caliber of the blood vessel, and a (Ll-second duration. Power is set at 400 mW and increased as necessary to look for blanching of the vessels for about 2 mm to 3 mm. Potential complications with this therapy include intracorneal hemorrhaging, iris atrophy, corneal thinning, recurrence of neovascularization, and corneal graft rejection.

Question 2 of 130 According to Eye Bank Association of America medical standards, which eye could not be used for corneal transplantation?

Eyes from a 75-year-old donor with 2200 cells/mm

An eye from a donor with diabetic retinopathy

An eye that has undergone phacoemulsification and has a normal endothelial cell count

An eye that has undergone excimer laser photorefractive keratectomy (PRK)

Please select an answer Feedback: Current Eye Bank Association of America standards allow for the use of tissue from older donors and eyes that have had previous intraocular surgery as long as the tissue rating itself is good. Diabetes may affect the corneal endothelium; hence it is a relative but not absolute contraindication for transplantation. Any corneal refractive surgery is a contraindication for use in corneal transplantation. The surgeon carries the ultimate responsibility for determining the suitability of transplant tissue for use in a particular recipient.

Page 2: Cornea/External Disease

Question 3 of 130 A 70-year-old man presents with a 10-day history of tearing, redness, and decreased vision of his right eye. He denies trauma. Slit-lamp examination reveals the findings in the figure. This man has a vascular necrosis of his conjunctiva, episclera, and sclera, with signs of acute inflammation. These signs are characteristic of necrotizing scleritis with inflammation. The workup and treatment of the patient should not include which of the following?

Referral to a rheumatologist for treatment with methotrexate or cyclophosphamide

High-dose oral prednisone

Review of systems focused particularly on autoimmune and infectious disorders

Topical corticosteroids

Please select an answer Feedback: In all cases of anterior scleritis, an underlying systemic cause should be sought. This is particularly true for necrotizing scleritis with inflammation, in which the 5-year mortality from autoimmune disease is approximately 30%. Disorders associated with scleritis include rheumatoid arthritis, as well as many other collagen vascular diseases. A complete review of systems should be obtained. Appropriate laboratory workup for an initial, mild episode of diffuse or nodular scleritis would include complete blood count, erythrocyte sedimentation rate, fluorescent treponemal antibody absorption (FTA-ABS), purified protein derivative (PPD) skin test, serum uric acid level, and other workup as indicated by the review of systems. In a necrotizing scleritis or any severe or recurrent scleritis, workup might also include c-ANCA (antineutrophilic cytoplasmic antibody), serum creatinine, rheumatoid factor, antinuclear antibody, and a chest x-ray. In necrotizing scleritis with inflammation, cultures to rule out primary or secondary infection should be considered as well. Because about half the patients who have necrotizing scleritis with inflammation lose vision from their disorder, treatment should be aggressive. The patient described would initially be treated with high-dose oral prednisone as well as some topical antibiotics. If cultures show the necrotic area to be sterile, then nonfortified topical antibiotic drops 2 to 4 times a day would be sufficient. There is some evidence that patients who have rheumatoid arthritis with necrotizing scleritis or scleritis with peripheral ulcerative keratitis have an improved life expectancy when treated with methotrexate or cyclophosphamide. These cytotoxic immunosuppressive agents take weeks to become effective, however, so the patient is also initially treated with high-dose oral prednisone. For ophthalmologists inexperienced in the use of immunosuppressive drugs, consultation with an internist or a rheumatologist is mandatory. Topical corticosteroids would not be expected to control the disease.

Page 3: Cornea/External Disease

Question 4 of 130 A 59-year-old man presents with a 10-year history of gradual painless vision loss of the right eye. There is a vague history of blunt trauma many years ago. The appearance of the eye is demonstrated in the figure below. Which one of the following would be the best therapeutic option to restore vision?

A. EDTA chelation therapy

B. Topical corticosteroids

C. Penetrating keratoplasty

D. Superficial keratectomy

Please select an answer Feedback: Salzmann's nodular degeneration is a disease of the anterior stroma. Hence, a full-thickness corneal transplantation would not be indicated. EDTA therapy is indicated for the treatment of calcific band keratopathy. Because Salzmann's represents scarring and not active inflammation, treatment with topical corticosteroids would be ineffective for this condition. The excimer laser could be used after mechanical keratectomy to remove anterior stromal scarring and smooth the underlying stroma, creating a new refractive surface. It would be very difficult, however, to completely remove the entire nodule with the excimer alone. Attempting to do so would result in severe irregular astigmatism due to different treatment ablation zones. A mechanical lamellar keratectomy alone could also be effective.

Question 5 of 130 What is the appropriate management of malignant melanoma of the conjunctiva?

Observation only, since there is a negligible mortality rate from these tumors

Excision with adjunct cryoablation

PUVA (psoralens plus ultraviolet A) therapy

Beta-irradiation

Please select an answer Feedback: Observation alone is not warranted in lesions of this nature. PUVA (psoralens plus ultraviolet A light) therapy has no place in the management of malignant melanoma, nor is beta-irradiation an appropriate modality. The optimal management of malignant melanoma has been controversial. The most common approach is combined excisional biopsy and adjunct cryoablation. Cryotherapy is selectively effective against melanocytic proliferation and appears to render recurrence less likely than if excision alone is used. The procedure should be performed using a no-touch technique under the operating microscope, with care taken to include 3 to 4 mm of adjacent normal conjunctiva. Double freeze-thaw applications are then applied to the surrounding conjunctiva. In exceptional cases of wide-spread malignant melanoma of the conjunctiva, orbital exenteration may be indicated.

Page 4: Cornea/External Disease

Question 6 of 130 A 70-year-old man presents with a 10-day history of tearing, redness, and decreased vision of his right eye. He denies trauma. Slit-lamp examination reveals the findings in the figure. Which one of the following is the most likely diagnosis?

Pseudomonas scleral ulcer

Nodular scleritis

Scleromalacia perforans

Necrotizing scleritis with inflammation

Please select an answer Feedback: This man has a vascular necrosis of his conjunctiva, episclera, and sclera, with signs of acute inflammation. These signs are characteristic of necrotizing scleritis with inflammation, which accounts for approximately 10% of all cases of anterior scleritis. Another 10% of patients with anterior scleritis have necrotizing scleritis without inflammation, also known as scleromalacia perforans. Most patients with scleromalacia perforans have long-standing rheumatoid arthritis, little sign of inflammation, and areas of thin, often staphylomatous sclera through which dark choroid can be seen. About 40% of patients with anterior scleritis have nodular scleritis, characterized by an inflamed, immobile, nodular thickening of the sclera. The remaining 40% of patients with anterior scleritis have diffuse scleritis. Diffuse scleritis is the most benign form and is often not associated with an underlying systemic condition. A Pseudomonas scleral ulcer would more likely be seen in the context of contact lens wear or trauma.

Page 5: Cornea/External Disease

Question 7 of 130 A 30-year-old woman presents with a red eye that is tender when she rubs it. The dilated vessels are deep and have a crisscross pattern, as demonstrated in the slit-lamp photograph. The inflamed vessels are not mobile when touched with a cotton-tipped applicator. Examination of the cornea, anterior chamber, and retina are normal, and the patient has no exoph-thalmos. This is the patient's first episode of a red, painful eye, and a review of systems for autoimmune and infectious diseases is negative. Which one of the following would be the best initial treatment?

Oral nonsteroidal anti-inflammatory agents, with meals

Subconjunctival injection of triamcinolone

High-dose oral prednisone and methotrexate

Topical prednisolone acetate 1%

Please select an answer Feedback: This patient has diffuse anterior scleritis. There is no sign of posterior scleritis. By a review of systems, it appears to be unassociated with systemic disease, but lab work to screen for treatable, underlying causes may be indicated. If this disorder proved unresponsive to treatment or was recurrent, further workup and consultation with a rheumatologist would probably be indicated. Currently, the recommended initial treatment for diffuse scleritis is an oral nonsteroidal anti-inflammatory agent, providing the patient has no renal disease. Topical steroids are generally ineffective with scleritis; injection of subconjunctival steroids has been shown to be effective, but these drugs carry an increased risk of local com-plications. Unless the patient's condition is unresponsive to the oral nonsteroidal agent, oral prednisone should not be used. With diffuse scleritis, use of cytotoxic immunosuppressive agents is generally reserved for patients who have peripheral ulcerative keratitis or patients requiring chronic oral prednisone to control their disease. In such patients, the risks of monitored immunosuppression with a cytotoxic agent may be less than the risks of chronic systemic prednisone therapy.

Question 8 of 130 What did the Collaborative Corneal Transplantation Studies find with regard to corneal transplant rejection?

Oral cyclosporin is effective in prolonging high-risk graft survival.

HLA-DR donor-host tissue matching decreased the incidence of graft failure.

An ABO blood match decreased the rate of graft failure in high-risk corneal transplants.

Preoperative cyclosporin is an important adjunct in the treatment of high-risk patients.

Please select an answer Feedback: The Collaborative Corneal Transplantation Studies (CCTS) were designed to evaluate the effect of donor-recipient histocompatibility matching and cross matching on the survival of corneal transplants in high-risk patients, defined as eyes with two or more quadrants of vascularization and/or a history of previous graft rejection. ABO blood group compatibility was determined but not used for recipient selection. HLA-A, HLA-B, or HLADR antigen matching did not

Page 6: Cornea/External Disease

significantly reduce the risk of graft failure. ABO blood group matching showed a small but significant reduction in the risk of graft failure. Cyclosporin was not studied in the CCTS.

Question 9 of 130 A 59-year-old man presents with a 10-year history of gradual painless vision loss of the right eye. There is a vague history of blunt trauma many years ago. The appearance of the eye is demonstrated in the figure below. Which one of the following is the most likely diagnosis?

Spheroidal degeneration

Band keratopathy

Salzmann's nodular degeneration

Herpes zoster keratitis

Please select an answer Feedback: This patient has the characteristic findings for Salzmann's nodular degeneration that is, bluish gray nodular elevations secondary to exuberant subepithelial scarring. Such scarring is usually secondary to some form of corneal injury, including trauma, dry eye, or infectious agents such as trachoma. Band keratopathy appears calcific and is typically located in the interpalpebral area spanning the 3 0'clock to 9 0'clock meridians. Spheroidal degeneration appears as fine golden or yellow deposits under the corneal epithelium and is usually the result of prolonged actinic exposure. The degenerative changes are located in the interpalpebral zone and the conjunctiva. Herpes zoster keratitis may present with epithelial "pseudodendrites." Late findings in zoster include a diffuse keratouveitis, disciform keratitis, or nummular infiltrates. Nodular elevations are not typical of zoster. Question 10 of 130 What is the primary mode of therapy for acne rosacea?

Topical retinoids

Oral tetracycline

Oral corticosteroids

Topical tetracycline ointment

Please select an answer Feedback: Oral tetracycline (250 mg bid) or oral doxycycline (100 mg bid) for 1 week, with subsequent tapering, appears to have a beneficial effect on sebaceous-gland dysfunction in rosacea. The use of either brings about improvement in both the ocular and the skin manifestations of the disease. Additional therapy should be directed to management of the blepharoconjunctivitis, including lid hygiene, the judicious use of low dose topical corticosteroids, and limiting the intake of spicy foods, caffeine, and alcohol.

Page 7: Cornea/External Disease

Question 11 of 130 A 42-year-old woman was cleaning a milk line on her farm with an undetermined substance. She accidentally splashed the substance into her left eye, resulting in immediate pain, redness, and loss of vision. The eye was immediately flushed with tap water and then, as soon as she arrived in the emergency room, with 2 liters of normal saline. She is later referred for evaluation. The slit-lamp appearance of the eye is shown in the figure. Signs present on examination that point to a poor prognosis would not include which of the following?

Elevated intraocular pressure

Symblepharon of the lower lid

Corneal edema

Conjunctival erythema

Please select an answer Feedback: Findings early in the course of a chemical burn can point to the overall prognosis for recovery of visual acuity. With mild chemical burns, the eyelids are generally unaffected, and chemosis and erythema of the bulbar and palpebral conjunctiva are minimal. The cornea may show superficial punctate keratitis. The intraocular pressure is usually normal. With moderate burns, the eyelids swell and the conjunctiva will have more chemosis with staining defects. The cornea will have epithelial defects and edema of the underlying stroma. In severe burns, the lid will be deformed due to damage and contracture of the underlying palpebral conjunctiva. Symblepharon may develop. The conjunctiva may often be ischemic. The cornea often becomes extremely edematous or opacified. As a result of secondary iritis or other anterior segment damage, the intraocular pressure may be elevated.

Page 8: Cornea/External Disease

Question 12 of 130 An 82-year-old woman underwent uncomplicated intracapsular cataract surgery 20 years ago. No intraocular lens implant was inserted. Her aphakic vision was good up until several years ago, when a gradual loss of vision developed. Her vision has now dropped to 20/100. The slit-lamp appearance of the eye is shown in the figure. Important details in her history and physical exam whould not include which of the following?

History of herpetic infection in this eye

Intact hyaloid face

Intraocular pressure

Amount of corneal astigmatism

Please select an answer Feedback: This patient has developed aphakic bullous keratopathy. Predisposing factors should be sought. If the patient had a history of herpetic infection involving the eye, then the corneal edema may be due to a reversible diffuse keratouveitis or immune disciform keratitis. Vitreous touching the endothelium often precipitates failure of the endothelial pump mecha-nism and should be identified. An intact hyaloid face that is well posterior to the endothelium might eliminate the need for a vitrectomy if surgical rehabilitation is pursued. An elevated intraocular pressure may also lead to corneal edema and may be difficult to identify in the face of stromal and epithelial edema with standard Goldmann applanation techniques. The amount of corneal astigmatism is not relevant to the development or treatment of aphakic bullous keratopathy.

Question 13 of 130 Which of the following statements does not accurately describe the role of acyclovir in the treatment of herpes zoster keratitis?

If given within 72 hours of the onset of skin lesions, it may reduce the incidence and severity of postherpetic neuralgia.

If given within 72 hours of the onset of skin lesions, it may reduce the duration of the skin lesions.

It may not be given concurrently with oral corticosteroids.

The proper dose is 600 mg to 800 mg orally 5 times daily for 10 to 14 days.

Please select an answer Feedback: Currently available topical antiviral agents effective against herpes simplex keratitis will not inhibit herpes zoster viral replication. Acyclovir given orally in a dosage of 600 to 800 mg 5 times daily for 10 to 14 days was shown in one study to reduce the incidence and severity of stromal keratitis and uveitis but not of postherpetic neuralgia. Another study demonstrated that 800 mg of oral acyclovir 5 times daily for only 7 days reduced the incidence of postherpetic neuralgia. Newer oral antiviral agents such as valacyclovir (Valtrex) and famcyclovir (Famvir) are better absorbed by the gastrointestinal system, resulting in higher blood levels; these newer agents may replace acyclovir as the drug of choice for

Page 9: Cornea/External Disease

this condition. Oral corticosteroids can be given with acyclovir and may reduce postherpetic neuralgia if given early in the course of the disease. Topical corticosteroids are used in the treatment of stromal keratitis, keratouveitis, or uveitis.

Question 14 of 130 A 72-year-old man is referred by his family physician for evaluation of decreasing vision and increasing erythema of the right eye over the last 24 hours. The patient also has had pain of the right scalp and several new "pimples" on his forehead. The appearance of the right eye is shown in the figure. Which of the following is the least likely ocular finding in this case?

Follicular conjunctivitis

Pars planitis

Myositis with diplopia

Iritis with ocular hypertension

Please select an answer Feedback: This patient most likely has herpes zoster of the ophthalmic division of the trigeminal nerve (herpes zoster ophthalmicus). The ipsilateral eye becomes involved about 50% of the time. When skin eruptions involve the tip of the nose (Hutchinson's sign), there is a greater likelihood of ocular involvement. Myositis may impede motility of the globe. A follicular reaction of the conjunctiva is common. The iridocyclitis or trabeculitis that may occur with zoster often results in elevation of the intraocular pressure. Other forms of secondary glaucoma are possible, including hemorrhagic glaucoma. The retina or optic nerve may become involved 2 to 3 weeks after the initial onset of the skin rash, but pars planitis has not previously been associated with this disorder.

Page 10: Cornea/External Disease

Question 15 of 130 Which statement would be least likely regarding the treatment of an alkali burn like the one shown?

Aids in healing the cornea might include bandage contact lenses, bilateral patching, 10% acetylcysteine, and clear plastic wrap.

Corneal transplantation may be considered 6 to 12 months following injury and only after any eyelid and conjunctival abnormalities are corrected.

Immediate irrigation of the eye with normal saline, lactated Ringer's solution, or tap water is mandatory.

Topical prednisolone should be continued for at least 1 month to ensure inhibition of detrimental inflammatory processes.

Please select an answer Feedback: The most important step in the initial treatment of chemical burns is complete and copious irrigation of the eye. Normal saline, lactated Ringer's solution, or even unsterile water (if that is the only liquid available) should be used. There should never be a delay for a better irrigant or for anything more than a cursory exam to confirm the globe is not lacerated from an explosive injury. The fornices should be swept with a cotton swab to ensure that no formed particles remain following the irrigation process. Litmus paper can be used as a guide in determining when enough irrigation has taken place. Movement toward a (neutral) pH of 7 10 minutes after 1-2 liters of liquid have been irrigated is a good sign that no further irrigation is necessary. In cases with corneal epithelial defects, therapeutic measures include pressure patching of the eyelids over an ophthalmic antibiotic ointment such as sulfacetamide or bandage soft contact lenses. The use of a moist chamber created with a clear plastic wrap may be beneficial in cases where the eyelids have been damaged and exposure is a problem. Early in the course of treatment, corticosteroids such as prednisolone are helpful in limiting damage due to inflammation; however, their collagenase activities may contribute to melting of the cornea. If topical corticosteroids are required for more than a week after the injury, topical progesterone should be considered to replace the corticosteroids. Corneal transplantation may be required to visually rehabilitate the eye, but it should not be undertaken until any eyelid or conjunctival problem is corrected. In most cases, it should not be performed for 6 to 12 months from the time of the initial injury. The prognosis for a successful transplant is very poor following alkali injuries because of the high risk of rejection. Limbal stem cell transplantation prior to penetrating keratoplasty may help improve the chance of maintaining a more normal epithelial surface postoperatively.

Page 11: Cornea/External Disease

Question 16 of 130 Important factors to consider in excimer laser phototherapeutic keratectomy does not include which of the following?

History of herpes simplex keratitis

Thickness of the cornea in the area of pathology

Refractive error of the fellow eye

ABO blood type

Please select an answer Feedback: Excimer laser phototherapeutic keratectomy (PTK) is quite effective when the pathology being treated is limited to the anterior 100 pm of the cornea in an area that is at least 400 pm thick. Because the cornea will become flatter after PTK, consideration of the fellow eye's refractive error is necessary. Excessive treatment may result in a large hyperopic shift secondary to corneal flattening, with intolerable anisometropia. The excimer laser can also cause the reactivation of latent herpes virus in the stroma. However, phototherapeutic keratectomy can be attempted in situations where the only other therapeutic option would be that of a penetrating keratoplasty. ABO blood typing is not a factor to consider when using the excimer laser.

Question 17 of 130 Which one of the following is the proper treatment for amiodarone-induced keratopathy?

Chelation treatment with EDTA

Observation alone

Observation with abrupt discontinuation of oral medication

Epithelial debridement

Please select an answer Feedback: Amiodarone deposition in the corneal epithelium, even when quite dense, rarely results in loss of vision. Deposits may occur as early as 2 weeks after starting the drug and probably occur in most patients after 4 months of therapy. Although the deposits may take up to a year to clear, the changes are generally reversible with cessation of the drug. Symptoms of glare at night with headlights may be reported. Amiodarone is used to reduce the risk of life-threatening dysrhythmias and should never be discontinued without consulting the patient's internist or cardiologist. Epithelial debridement will only temporarily remove the opacification. EDTA therapy is a treatment for calcific band keratopathy and not an appropriate therapeutic option in this type of keratopathy.

Page 12: Cornea/External Disease

Question 18 of 130 A 46-year-old migrant worker presents with a sore right eye. A slit-lamp examination reveals the corneal appearance as shown in the figure. He had surgery performed for this process several years ago. Which of the following is not an indication for surgery?

Documentation of recent growth now approaching the visual axis

Chronic inflammation and irritation

3 mm of growth onto the cornea with the presence of a Stocker line

Medial rectus restriction with diplopia on side gaze

Please select an answer Feedback: This patient has a recurrent pterygium beginning to encroach on the visual axis. It is relatively inflamed. Recurrent pterygia are generally more aggressive than primary pterygia. In general, indications for surgery include documentation of growth that is threatening the visual axis, an induced strabismus, inflammation or irritation unresponsive to medical treatment, or dellen formation. Irregular or regular astigmatism (generally, with the rule) caused by a pterygium may also be an indication for surgery. The mere presence of a pterygium is not an indication for surgery because of the risk of recurrence. A Stocker line is a deposit of iron in front of the pterygium. It indicates little recent growth and is not an indication for surgery. Some may argue that cosmesis is an indication for surgery.

Page 13: Cornea/External Disease

Question 19 of 130 A 52-year-old man presents for a routine eye examination. He is asymptomatic with a visual acuity of 20/20 OU. The slit-lamp appearance of his left cornea is shown in the figure. What is the most likely cause for this finding?

Amiodarone

Topical epinephrine ophthalmic solution

Ochronosis

Chronic active hepatitis

Please select an answer Feedback: Corneal verticillata, also known as vortex keratopathy, may be caused by various medications (including amiodarone, indomethacin, chloroquine, and chlorpromazine) as well as by Fabry's disease (X-linked alphagalactosidase deficiency). Amiodarone is used to treat patients with cardiac dysrhythmias. Deposits in the epithelial layer may create a whorl-like pattern. Fabry's disease is characterized by renal failure, peripheral neuropathy, and hyperkeratotic skin lesions. Epinephrine usually causes dark pigmentation of the conjunctiva (adrenochrome deposits), not the cornea. The pigmentation that occurs with ochronosis is generally in the anterior corneal stroma as well as in the sclera and episclera. Chronic active hepatitis may result in elevated serum copper levels that present as pigmentation in the periphery of the cornea at the level of Descemet's membrane; this is similar to the Kayser-Fleischer ring seen in patients with Wilson's disease (hepatolenticular degeneration).

Page 14: Cornea/External Disease

Question 20 of 130 A 92-year-old woman seeks consultation for a sore left eye. The pain is described both as a foreign-body sensation as well as a deep aching pain. On further examination, it is determined that the patient's visual acuity is bare LP with poor projection. The intraocular pressure is 40 mm Hg. Visual acuity in her right eye is 20/30. A 3.0 log unit relative afferent pupil-lary defect is present in the left eye, and the optic nerve is pale, with a .99 cup-to-disc ratio. Her main complaint at this time is pain. The appearance of the left eye on external examination with the slit lamp is shown below. Which one of the following would be the least appropriate long-term therapeutic option in this particular case?

Conjunctival flap

Topical glaucoma medication

Bandage contact lens

Stromal cauterization

Please select an answer Feedback: The visual potential for this patient is quite limited because of end-stage glaucoma. In such instances, therapy should be directed toward pain relief rather than visual restoration. A bandage contact lens would reduce the pain from the bullae, but long-term use on a decompensated cornea carries a high risk of corneal infection. Lowering the intraocular pressure could eliminate both the deep pain and the bullae that are producing the foreign-body sensation. Cauterization of Bowman's membrane and the anterior stroma can be attempted. Diathermy is applied to the anterior stroma to create a barrier limiting the degress of fluid from the stroma to the epithelium, thus decreasing the tendency of bullae formation. It may also help the epithelium adhere to the stroma more securely, thus resisting blister formation. If stromal cauterization fails, a conjunctival flap, as described by Gundersen, can be performed. Possible problems with the operation include the potential for a less cosmetically acceptable appearance, decreased view of the anterior segment, and less penetration into the anterior segment of topical ophthalmic medications.

Page 15: Cornea/External Disease

Question 21 of 130 A 22-year-old man complains of tearing, photophobia, and the appearance of a white spot in the center of his cornea for 1 day. He wore contact lenses until the onset of his symptoms. When examined, he is noted to have a markedly thickened central cornea with opacification. In the other eye, the cornea is thinned centrally and there is slight apical scarring. The history and signs are most consistent with which diagnosis?

Traumatic abrasion

Contact lens-induced corneal edema

Bacterial corneal abscess

Acute hydrops in keratoconus

Please select an answer Feedback: The sudden development of corneal edema, referred to as acute hydrops, results from a break in Descemet's membrane in advanced keratoconus. The most common presentation is tearing, photophobia, and marked corneal edema of sudden onset.

Question 22 of 130 Which of the following is not an indication for surgery in keratoconus?

The inability to retain a contact lens

Progressive against-the-rule astigmatism

A poor best-corrected vision with an optimally fit contact lens

Excessive apical or peripheral corneal thinning

Please select an answer Feedback: Against-the-rule astigmatism is not a characteristic sign of keratoconus. High astigmatism may be part of the early development of the disease, but as thinning and scarring progress, irregular astigmatism becomes the hallmark of the disease. First-line treatment consists of fitting with either a standard contact lens or a contact lens specially designed for keratoconus. When the patient no longer has functional vision even with the best-fit contact lens or when a contact lens is not functionally retainable because of the curvature of the cornea, penetrating keratoplasty is the surgical treatment of choice. Other surgical approaches include thermokeratoplasty and lamellar keratoplasty (epikeratoplasty).

Question 23 of 130 Which of the following would not produce crystalline deposits in the cornea?

Chlorpromazine

Systemic paraproteinemias

Streptococcal infection

Cystinosis

Please select an answer Feedback: Crystalline deposits in the cornea may be produced by systemic metabolic abnormalities, such as cystinosis. In this condition, multiple refractile crystals are deposited throughout the cornea and conjunctiva. These findings are

Page 16: Cornea/External Disease

associated with both the infantile nephropathic form as well as the benign adult form of cystinosis. Systemic paraproteinemias, such as multiple myeloma, may also be characterized by crystalline corneal deposits. Streptococcal infection in the form of crystalline keratopathy may also manifest as crystalline deposition, as mentioned in Question E39. Other causes of crystalline keratopathy include cholesterol deposition, Schnyder's crystalline dystrophy, calcium oxalate (Diejjenbachia plant sap exposure), Bietti's marginal crystalline dystrophy, other drugs such as indomethacin, chloroquine, and thioridazine; gout, and uremia. Chlorpromazine in higher doses for long periods produces a brownish tan powder like deposit in the deep stroma of the cornea; the deposition is not, however, crystalline.

Question 24 of 130 A 21-year-old man has had a red, irritated right eye for 4 weeks. He states that he was seen initially in an outpatient emergency clinic after 1 week of symptoms and was given antibiotic drops, which he took 4 times a day for 10 days. Despite using the eye drops, he has noticed an increase in the discharge from his right eye, and in the past 2 days his left eye has become red and irritated and has developed a yellow mucoid discharge. He complains of only mild blurring of vision in the right eye. Examination reveals a coarse punctate epithelial keratitis and bilateral follicular conjunctivitis. Which of the following types of conjunctivitis is least likely to be associated with the patient's signs and symptoms?

Chlamydia

Allergic

Bacterial

Viral

Please select an answer Feedback: Acute follicular conjunctivitis can be associated with a variety of causes. The differential diagnosis includes adenoviral conjunctivitis, primary herpes simplex, toxic conjunctivitis (medicamentosa or secondary to molluscum contagiosum), adult inclusion (chlamydial) disease, acute hemorrhagic conjunctivitis, and Newcastle's disease. Certain bacterial pathogens also may produce a follicular conjunctival response. Allergic conjunctivitis is more commonly associated with a nonspecific papillary response.

Question 25 of 130 Which of the following is not a treatment for Fuchs' dystrophy?

Penetrating keratoplasty

Bandage contact lenses

Deep anterior lamellar keratoplasty

Sodium chloride drops and ointment

Please select an answer Feedback: Deep anterior lamellar keratoplasty has no role in the management of Fuchs' dystrophy, which is primarily an endotheliopathy. Hypertonic agents may be used to minimize symptoms of microcystic and bullous edema. Similarly, bandage lenses can be used to provide temporary relief of pain in the patient with painful bullous keratopathy. If improvement of vision is not a goal in the patient with painful bullous keratopathy, corneal cautery or a conjunctival flap may be employed to relieve symptoms. Although Penetrating keratoplasty has been the definitive surgery for the management of Fuchs' dystrophy, in recent years, endothelial keratoplasty has become the preferred surgical treatment.

Page 17: Cornea/External Disease

Question 26 of 130 Which of the following is not included in the management of recurrent corneal erosion?

Partial-depth anterior stromal puncture

Hypertonic drops and ointments

Patching or bandage lenses

Chemical cautery

Please select an answer Feedback: Recurrent erosion of either dystrophic or traumatic origin can be treated with lubrication, application of hypertonic sodium chloride, therapeutic bandage lenses, pressure patching, or anterior stromal puncture.

Question 27 of 130 Which of the following is not a sign associated with keratoconus?

Early scissoring reflex on retinoscopy

Apical scarring

Fine vertical folds in pre-Descemet's membrane stroma

Epithelial copper deposition

Please select an answer Feedback: Keratoconus is associated with a characteristic iron deposit in the corneal epithelium (Fleischer's ring), which is located at the base of the cone. A scissoring of the retinoscopic reflex is one of the earliest signs of keratoconus and may precede obvious changes in the curvature and thickness of the cornea. Advanced cases of keratoconus demonstrate central corneal thinning and apical scarring. Frequently, there are fine vertical folds (Vogt's striae) and, later in the disease, reticular scarring of the anterior stroma. Keratoconus demonstrates inferior steepening at the keratometer; photokeratoscopy or topographic analysis demonstrates characteristic changes.

Question 28 of 130 Which therapy is the least appropriate for keratitis sicca?

Punctal occlusion

Tear substitutes

Application of mucolytic agents

Topical antibiotics

Please select an answer Feedback: Standard initial therapy of dry eye begins with the application of artificial tears. Tear substitutes alone, however, may not be adequate to control surface keratopathy. In such cases, occlusion of either two or all four of the puncta may be an effective adjunct to management. Mucolytic agents, such as 10% N-acetylcysteine, applied 2 to 3 times daily may be helpful in reducing symptoms from mucous plaques on the corneal surface. Topical corticosteroids and other anti-inflammatory agents such as cyclosporine have also been used with good successes. Unless there is an epithelial defect, topical antibiotics are unnecessary in treating dry eyes.

Page 18: Cornea/External Disease

Question 29 of 130 Which of the following statements best describes Fuchs' dystrophy?

Topical corticosteroids are useful in the treatment of early Fuchs' dystrophy.

Fuchs' dystrophy is associated with elevated intraocular pressure in 65% of cases.

Symptoms may include pain and decreased vision.

Specular microscopy of the patient with Fuchs' dystrophy demonstrates vesicular changes.

Please select an answer Feedback: Symptoms of Fuchs' dystrophy may include decreased vision from stromal and epithelial edema, as well as pain from epithelial breakdown in more advanced stages of the disease. Corticosteroids have no beneficial effect in the early stages of this dystrophic, noninflammatory disorder. Although lowering the intraocular pressure in the patient with an edematous cornea from Fuchs' dystrophy may temporarily help the edema, elevated intraocular pressure is not a consistent feature of the disorder. Specular microscopy early in the disease demonstrates pleomorphism, polymegathism, and a disruption of the endothelial monolayer by Descemet's membrane excrescences. Vesicular endothelial changes are seen more characteristically in posterior polymorphous dystrophy.

Question 30 of 130 What is the treatment of choice for gonococcal conjunctivitis?

Intramuscular ceftriaxone and topical ocular saline irrigation

Topical fortified penicillin or cephalosporin

Bacitracin ointment and fortified topical gentamicin

Topical and systemic tobramycin

Please select an answer Feedback: Gonococcal conjunctivitis requires maximum systemic antibiotic therapy. Topical therapy may be added but is of secondary importance. Ocular irrigation may be of assistance in removing infectious agents from the ocular surface. Because of the increasing problem of penicillin resistant strains of gonococcus in certain areas of the United States, ceftriaxone administered intramuscularly, 1 gram daily for 5 consecutive days, is now the drug of choice. If the organism is highly sensitive to penicillin and the patient does not have a history of penicillin allergy, penicillin becomes the antibiot ic of first choice.

Page 19: Cornea/External Disease

Question 31 of 130 A 40-year-old woman presents with a history of several days of redness in a single sector of her left eye, as demonstrated in the figure. She also has moderately deep pain of the involved eye. She denies foreign-body sensation. She has had no previous eye problems. On examination, most of the dilated vessels are radial and can be moved with a cotton tip applicator. These vessels blanch with instillation of topical Neo-Synephrine 2.5%. The overlying conjunctiva also has some dilated vessels, but its epithelial surface is normal. A review of systems for systemic autoimmune and infectious diseases is negative. Which of the following would be communicated to the patient?

She requires an extensive rheumatologic workup.

She should avoid use of birth control pills.

Recurrences may lead to some permanent visual loss.

Further workup is not necessary at this time, but her condition is likely to recur.

Please select an answer Feedback: Episcleritis is often recurrent. About one-third of all cases of episcleritis are associated with systemic immunologic or infectious disease, but episcleritis is very rarely the initial manifestation of the disease. Laboratory workup is generally not necessary unless indicated by a review of systems or unless the episcleritis is severe, recurrent, or difficult to distinguish from scleritis. Some patients with episcleritis have posterior blepharitis and eventually present with a sector of inflamed episclera associated with a 1- to 2-mm infiltrate near the limbus. Therefore, some cases initially diagnosed as episcleritis are actually staphylococcal marginal disease. Episcleritis does not result in loss of visual function. The use of oral contraceptives has not been shown to be associated with episcleritis.

Page 20: Cornea/External Disease

Question 32 of 130 A 40-year-old woman presents with a history of several days of redness in a single sector of her left eye, as demonstrated in the figure. She also has moderately deep pain of the involved eye. She denies foreign-body sensation. She has had no previous eye problems. On examination, most of the dilated vessels are radial and can be moved with a cotton tip applicator. These vessels blanch with instillation of topical Neo-Synephrine 2.5%. The overlying conjunctiva also has some dilated vessels, but its epithelial surface is normal. Which of the following diagnoses is most likely?

Episcleritis

Squamous cell carcinoma of the conjunctiva

Nodular scleritis

An inflamed pinguecula

Please select an answer Feedback: In this patient the inflammation appears to be in the episcleral tissues, so she has episcleritis. Episcleritis is classified as simple or nodular. Simple episcleritis usually presents as a sector of inflammation, sparing most of the remaining episclera. In nodular episcleritis, the inflammation is accompanied by a mobile nodule. In scleritis, the deeper scleral vessels are more involved than the episclera, so the dilated vessels show a crisscross pattern and are not mobile. A pinguecula may become inflamed but usually is not associated with pain other than foreign-body sensation. Pingueculae are typically located in the nasal or temporal bulbar conjunctiva, whereas the inflamed tissue in this case is in an oblique meridian. A squamous cell carcinoma often has dilated feeder vessels, but there is usually no pain, and the conjunctival epithelium appears abnormal.

Question 33 of 130 Which of the following statements is the least accurate about adult inclusion disease?

The patient's sexual partners should be seen by a gynecologist, urologist, or other appropriate physician.

Treatment of the patient's sexual partners includes systemic medication.

Treatment of the patient's sexual partners with topical medication is adequate.

It is important to rule out other diseases, such as syphilis or gonorrhea, in the patient's sexual partners.

Please select an answer Feedback: If the diagnosis of adult inclusion disease is established, the patient's sexual partners should be treated systemically to prevent reinfection or the spread of disease. This is particularly true if a cervicitis or urethritis is suspected in the patient or the patient's sexual partners. The consulting gynecologist or urologist may wish to rule out other venereally transmitted diseases before treating the patient's sexual partners with oral tetracycline or erythromycin.

Page 21: Cornea/External Disease

Question 34 of 130 Which of the following statements does not describe herpes simplex?

Herpes simplex type 1 is more commonly associated with oral, labial, and ocular disease.

Herpes simplex virus can be isolated from geographic (ameboid) herpes simplex keratitis.

About 15% of the US population carries systemic antibodies to the herpes simplex virus.

Primary herpes simplex keratoconjunctivitis can present as an acute follicular conjunctivitis without skin lesions.

Please select an answer Feedback: Herpes simplex virus is ubiquitous; hence, the vast majority (90%) of the US population is exposed to the herpes simplex virus during childhood and carries systemic antibodies, although most encounters are subclinical. As opposed to type 1, herpes simplex type 2 is more commonly transmitted venereally. This does not, however, mean that type 2 never causes ocular lesions. Question 35 of 130 The treatment of acute graft rejection includes the administration of which of the following?

Blocking antibodies

Cytotoxic agents

Intensive topical, periocular, and, occasionally, systemic corticosteroids

Nonsteroidal anti-inflammatory agents

Please select an answer Feedback: Intensive corticosteroid therapy is the mainstay of treatment of the graft rejection. Increased topical therapy may be sufficient for epithelial rejection or subepithelial infiltrates. If clinical edema and/or a rejection (Khodadoust) line is present, periocular injection and/or systemic corticosteroid administration may be warranted. Question 36 of 130 Which of the following is not part of the differential diagnosis of chronic follicular conjunctivitis?

Herpes simplex

Parinaud's oculoglandular syndrome

Trachoma

Toxic conjunctivitis

Please select an answer Feedback: The causes of follicular conjunctivitis lasting longer than 4 weeks include adult inclusion disease, trachoma, toxic conjunctivitis, Parinaud's oculoglandular syndrome, Axenfeld's conjunctivitis, Thygeson's superficial punctate keratitis, acne rosacea, and benign lymphoid hyperplasia. Faced with a patient who has chronic follicular conjunctivitis, the ophthalmologist should consider these additional entities. Therefore, in addition to obtaining a sexual history, the ophthalmologist should inquire about the use of eye or facial cosmetics, skin lesions, foreign travel, and contact with small animals. Herpes simplex may indeed be the cause of follicular conjunctivitis. However, it is usually self-limited or recurrent rather than chronic.

Page 22: Cornea/External Disease

Question 37 of 130 A Giemsa-stained scraping from a patient with acute adult inclusion disease is likely to show which of the following?

A mixture of polymorphonuclear leukocytes and lymphocytes

Nnumerous intracytoplasmic inclusions

A preponderance of lymphocytes

A preponderance of polymorphonuclear leukocytes

Please select an answer Feedback: Scrapings from patients with new-onset adult inclusion disease characteristically show a mixed polymorphonuclear and lymphocytic response and may include plasma cells, immature leukocytes, and giant cells. A preponderance of lymphocytes is more consistent with an established viral infection, whereas a predominant polymorphonuclear response is more common with an acute bacterial infection. Although basophilic paranuclear intracytoplasmic inclusion bodies are specific for chlamydial infection, such inclusions are not commonly found in cytologic scrapings of adult inclusion disease. In the neonatal counterpart of adult inclusion disease (inclusion blennorrhea), these intracytoplasmic inclusions are much more commonly found. Diagnosis on the basis of cytologic staining requires meticulous scrutiny of the scraping.

Question 38 of 130 A 40-year-old woman presents with a history of several days of redness in a single sector of her left eye, as demonstrated in the figure. She also has moderately deep pain of the involved eye. She denies foreign-body sensation. She has had no previous eye problems. On examination, most of the dilated vessels are radial and can be moved with a cotton tip applicator. These vessels blanch with instillation of topical Neo-Synephrine 2.5%. The overlying conjunctiva also has some dilated vessels, but its epithelial surface is normal. The best treatment would include which one of the following?

Acetaminophen and cold compresses

Resection of the lesion

A topical antibiotic

Topical corticosteroids or an oral nonsteroidal anti-inflammatory agent

Please select an answer Feedback: Many cases of episcleritis are self-limited. As this patient has moderate pain, treatment is indicated. This disorder usually responds well to a course of topical corticosteroids or an oral nonsteroidal anti-inflammatory agent. As this process is not due to an infectious agent, a topical antibiotic would be inappropriate. Resection of the lesion is not indicated. Acetaminophen has not been shown to be an effective treatment of this condition.

Page 23: Cornea/External Disease

Question 39 of 130 Which of the following disorders is not caused by chlamydial organisms?

Molluscum contagiosum

Lymphogranuloma venereum

Endemic trachoma

Neonatal conjunctivitis

Please select an answer Feedback: Although molluscum contagiosum can be venereally transmitted, it is the result of a pox virus. Endemic trachoma, a leading cause of blindness worldwide, is caused by serotypes A to C of Chlamydia trachomatis. Adult inclusion disease is generally associated with serotypes 0 to K, and lymphogranuloma venereum with the L serotype of Chlamydia. Neonatal conjunctivitis of chlamydial origin may occur in 2% to 6% of the neonates in the United States. It presents as a mucopurulent conjunctivitis in the newborn. Inflammation may be intense and can result in corneal micropannus and scarring if untreated. Inclusion conjunctivitis of the newborn may be associated with systemic chlamydial infection, including otitis, rhinitis, and pneumonitis, and requires systemic treatment.

Question 40 of 130 A case of recurrent pterygium is shown in the figure. The patient whose eye is shown had surgery several years ago. Which of the following surgical treatments would be the least effective in this case?

Conjunctival sliding flap

Bare scleral resection alone

Conjunctival autograft at the time of bare scleral resection

Resection with intraoperative application of mitomycin to the scleral bed

Please select an answer Feedback: In cases of a recurrent pterygium, especially those as severe as in this patient, bare scleral resection alone will almost certainly result in the recurrence of another large pterygium. A case can be made for treating primary as well as recurrent pterygia with adjunctive therapy. Topical mitomycin as well as conjunctival autografting or an advancement conjunctival flap have been shown to significantly reduce the rate of recurrence. However, there have been reports of scleral melts with topical mitomycin. Intraoperative mitomycin application followed by a pedicle flap to cover the area of treated sclera should be considered if resection with a conjunctival autograft has failed.

Page 24: Cornea/External Disease

Question 41 of 130 Enlarged corneal nerves can be seen in which one of the following conditions?

Fuch's corneal dystrophy

Familial dysautonomia

Keratoconus

Ichthyosis

Please select an answer Feedback: Familial dysautonomia (Riley-Day syndrome), multiple endocrine neoplasia type IIB, Phytanic acid storage disease, leprosy, and neurofirbromatosis can all be associated with enlarged corneal nerves. In keratoconus, ichthyosis, and Fuch's corrneal dystrophy, the corneal nerves are more visible, but they are actually not enlarged.

Question 42 of 130 In healthy adults, which of the following is usually considered to be a part of the normal flora of the ocular surface?

Serratia marcescens

Fusarium solani

Escherichia coli

Staphylococcus epidermidis

Please select an answer Feedback: Staphylococcus epidermidis is the most commonly isolated bacteria on the normal ocular surface. Other organisms that are considered to be a part of the normal ocular flora in adults include other coagulase-negative staphylococci, S. aureus, and diphtheroids. Escherichia cole is normal only in infants following vaginal birth. Both Serratia and Fusarium species are pathogenic and not a normal part of the ocular flora.

Question 43 of 130 HIV infection is a risk factor for the development of which one of the following?

Conjunctival intraepithelial neoplasia

Conjunctival melanoma

Sebaceous gland carcinoma

Neurofibroma

Please select an answer Feedback: Conjunctival intraepithelial neoplasia most commonly develops on exposed areas of the bulbar conjunctiva, and HIV infection appears to be associated with a higher incidence as well as more rapid growth. Neurofibromas are subconjunctival peripheral nerve sheath tumors, and it is almost always a manifestation of neurofibromatosis. Conjunctival melanomas may arise from acquired nevi, primary acquired melanosis, or normal conjunctiva. Sebaceous gland carcinomas of the eyelid often have a multifocal nature, therefore environmental factors leading to malignant transformation are likely at play.

Page 25: Cornea/External Disease

Question 44 of 130 A prominent anteriorly displaced Schwalbe's line with attached iris strands, iris hypoplasia, and glaucoma are characteristic of which one of the following?

Sclerocornea

Circumscribed posterior keratoconus

Axenfeld-Rieger syndrome

Peter's anomaly

Please select an answer Feedback: Axenfeld-Rieger syndrome represents a spectrum of disorders characterized by an anteriorly displaced Schwalbe's ring with attached iris strands, iris hypoplasia, and glaucoma in 50% of cases occuring in late childhood or in adulthood. Also often present are associated skeletal, cranial, facial, and dental abnormalities. Peter's anomaly is a central corneal opacity that may be associated with variable degrees of iridocorneal adhesion extending from the region of the iris collarette to the cornea. This is present at birth, and systemic malformations including developmental delay, heart defects, external ear abnormalities, hearing loss, CNS defects, aspinal defects, gastrointestinal and genitourinary defects, facial clefts, and skeletal abnormalities are present in up to 60% of patients. Circumscribed posterior keratoconus is characterized by a localized central or paracentral indentation of the posterior cornea without a protrusion of the anterior surface. Sclerocornea is a nonprogressive, noninflammatory scleralization of the cornea that can involve the entire cornea. Multiple systemic abnormalities have been reported with this condition.

Question 45 of 130 The inheritance pattern of which one of the following corneal dystrophies is X-linked dominant?

Epithelial basement membrane dystrophy

Corneal dystrophy of Bowman layer

Meesmann dystrophy

Lisch corneal dystrophy

Please select an answer Feedback: Lisch corneal dystrophy was first described in 1992, and it was found to be phenotypically and genetically distinct from Meesmann dystrophy. The inheritance pattern is X-linked dominant. Epithelial basement membrane dystrophy may have a dominant inheritance pattern with incomplete penetrance, and it is often sporadic. Meesmann dystrophy and Corneal dystrophy of Bowman layer (formerly Reis-Bucklers dystrophy and Thiel-Behnke dystrophy) are both autosomoal-dominant dystrophies.

Page 26: Cornea/External Disease

Question 46 of 130 Acanthamoeba and fungi are best seen in tissue with which stain?

Calcofluor white

Gram

Lectin

Acid-fast

Please select an answer Feedback: Both calcofluor white and acridine orange are useful stains for identifying acanthamoeba and fungi. Fungi can also be identified on Gram stain. Aerobic and anaerobic bacteria can be seen on Gram stain and with acridine orange. Mycobacteria are seen with Gram, acid-fast, and lectin staining.

Question 47 of 130 Acanthamoeba and fungi are best seen in tissue with which stain?

Calcofluor white

Gram

Lectin

Acid-fast

Please select an answer Feedback: Both calcofluor white and acridine orange are useful stains for identifying acanthamoeba and fungi. Fungi can also be identified on Gram stain. Aerobic and anaerobic bacteria can be seen on Gram stain and with acridine orange. Mycobacteria are seen with Gram, acid-fast, and lectin staining.

Question 48 of 130 Parinaud oculoglandular syndrome is characterized by which of the following?

Chronic recurrent conjunctivitis

Granulomatous conjunctivitis with regional lymphadenopathy

Acute conjunctivitis with pseudomembranes

Cicatrizing conjunctivitis

Please select an answer Feedback: Parinaud oculoglandular syndrome is characterized by a unilateral conjunctivitis with 1 or more rasied or flat, gelatinous granulomatous lesions on the tarsal, forniceal, or bulbar conjunctiva that is temporally associated with (either simultaneously or shortly followed by) the development of a unilateral firm and tender preauricular and submandibular lymphadenopathy. Occasionally, cervical nodes develop. The causative agent in most cases is Bartonella henselae (from cat-scratch disease), but other causes include tularemia, tuberculosis, sporotrichosis, syphilis, and coccidioidomycosis. Chronic recurrent conjunctivitis without associated lymphadenopathy can be caused by many types of bacterial organisms, especially those that can colonize the nasal passages, sinuses, or lacrimal drainage system (chronic dacrocystitis). Cicatrizing conjunctivitis is characterized by subepithelial fibosis and scarring of the conjunctiva, often associated with

Page 27: Cornea/External Disease

symblepharon formation. This can be seen in cases of mucous membrane pemphigoid, Stevens-Johnson syndrome, sarcoidosis, toxic epidermal necrolysis, epidemic keratoconjunctivitis, chemical burns, and pseudopemphigoid. Acute conjunctivitis with pseudomembranes can frequently be seen in severe cases of viral conjunctivitis with adenovirus, but also in cases of herpes simplex virus conjunctivitis.

Question 49 of 130 Plasminogen deficiency is thought to be the etiology for which one of the following?

Ligneous conjunctivitis

Granulomatous conjunctivitis

Atopic dermatitis

Vernal keratoconjunctivitis

Please select an answer Feedback: Ligneous conjunctivitis is a rare diorder characterized by the formation of firm "woody" yellowish fibrinous pseudomembranes on the conjunctival surface. These membrane are composed of fibrin, epithelial cells, and inflammatory cells. The cause of ligneous conjunctivitis is secondary to severe type I plasminogen deficiency. Vernal keratoconjunctivitis is a seasonal, recurrent, inflammatory condition that occurs predominantly in boys. The condition invovles both types I and IV hypersensitivity reactions. The conjunctival infiltrate consists of various inflammatory cells. Granulomatous conjunctivit is with regional lymphadenopathy (Parinaud oculoglandular syndrome) is characterized by unilateral conjunctivitis with 1 or more raised or flat gelatinous lesions on the conjunctiva. This condition is most commonly caused by infection with Bartonella henselae. Atopic dermatitis is a chronic condition that is caused by environmental irritants, climate, and psychological factors in susceptible individuals. Immunologic changes include increased IgE hypersensitivity, increased histamine release, and imparied cell-mediated immunity.

Question 50 of 130 Bluish-white subepithelial elevations located in the paracentral zone of the cornea most likely represents which one of the following?

Corneal keloid

Salzmann nodular degeneration

Lipid keratopathy

Amyloid degeneration

Please select an answer Feedback: Salzmann nodular degeneration is a noninflammatory corneal degeneration that sometimes results from old, long-standing keratitis. These nodules are gray-white or blue-white in appearance, elevated, and may be associated with recurrent erosions. Histologically, these lesions consist of hyaline and fibrillar material in place of Bowman's layer. Amyloid degeneration may also be associated with corneal inflammation, or trauma, but these deposits are raised yellow-pink nodular masses in the cornea. Corneal keloids form in response to corneal perforation or injury. They are white, protuberant masses that can resemble dermoids. Lipid keratopathy results after prolonged corneal inflammation. The deposits are yellow or cream-colored in areas of vascularized scars.

Page 28: Cornea/External Disease

Question 51 of 130 What would a workup for episcleritis include?

Complete blood count

Conjunctival biopsy

C-reactive protein test

Temporal artery biopsy

Please select an answer Feedback: Some patients with an unremarkable review of systems may benefit from a limited workup. This includes patients with nodular episcleritis or those with severe and recurrent/persistent simple episcleritis. Useful laboratory studies in this group of patients include serum uric acid, complete blood count with differential, antinuclear antibody, rheumatoid factor, erythrocyte sedimentation rate, Venereal Disease Research Laboratory (VDRL) test, fluorescent treponemal antibody absorption (FTA-ABS) test, and chest x-ray.

Question 52 of 130 What is a common complication of tarsorrhaphy?

Ectropion

Corneal melting

Trichiasis

Microbial keratitis

Please select an answer Feedback: Tarsorrhaphy is an effective method for treating nonhealing epithelial defects. Complications of tarsorrhaphy include trichiasis, premature opening of the tarsorrhaphy, pyogenic granuloma, and keloid formation.

Question 53 of 130 Which of the following is a pertinent clinical feature of Keratoglobus?

Focal corneal thinning and protrusion

Corneal plana

Diffuse guttata

Limbus to limbus corneal thinning

Please select an answer Feedback: Keratoglobus is characterized by diffuse corneal thinning, not a focal abnormaility. Cornea plana is a flat cornea often associated with hyperopia. Diffuse gutatta are a hallmark of Fuchs' corneal dystrophy.

Page 29: Cornea/External Disease

Question 54 of 130 Which of the following factors does not need to be considered in excimer laser phototherapeutic keratectomy?

Refractive error of the fellow eye

ABO blood type

Thickness of the cornea in the area of pathology

History of herpes simplex keratitis

Please select an answer Feedback: Excimer laser phototherapeutic keratectomy (PTK) is quite effective when the pathology being treated is limited to the anterior 100 u of the cornea in an area that is at least 400 u thick. Because the cornea will become flatter after PTK, consideration of the fellow eye' s refractive error is necessary. Excessive treatment may result in a large hyperopic shift secondary to corneal flattening, with intolerable anisometropia. The excimer laser can also cause the reactivation of latent herpes virus in the stroma. However, phototherapeutic keratectomy can be attempted in situations where the only other therapeutic option would be that of a penetrating keratoplasty. ABO blood typing is not a factor to consider when using the excimer laser.

Question 55 of 130 Which of the following is a complication of prolonged treatment with topical trifluridine?

Cataract

Elevated intraocular pressure

Corneal stromal melting

Punctal stenosis

Please select an answer Feedback: Topical trifluridine is reported to cause epithelial keratopathy, burning, irritation, hyperemia, stromal edema, palpebral edema, superficial punctate keratitis, increased IOP, and punctal stenosis.

Question 56 of 130 Which of the following findings is not caused by staphylococcal blepharitis?

Corneal phlyctenules

Chronic follicular conjunctivitis

Marginal corneal infiltrates

Meibomian plugging

Please select an answer Feedback: Chronic colonization of the lid margin with pathogenic staphylococci may be associated with a number of secondary phenomena. Phlyctenules may be seen on the cornea, straddling the limbus, or on the conjunctiva and, like marginal corneal infiltrates, represent an immune response to staphylococcal antigens. Staphylococcal blepharitis may induce a nonspecific follicular response in the conjunctiva. Plugging or inspissation of the meibomian glands, however, is

Page 30: Cornea/External Disease

more commonly seen with seborrheic blepharitis than with staphylococcal disease. The latter is characterized by hyperemia of the lid margin, marginal ulcerations, and collarette formation.

Question 57 of 130 Which statement best describes the use of antiviral agents for the management of herpes simplex infection?

They are always preferable to epithelial debridement

They should be used in combination with broad-spectrum antibiotics

They are augmented by concomitant corticosteroid use

They may have direct toxic effects on the epithelium with prolonged use

Please select an answer Feedback: Topical antiviral agents may have direct toxic effects on the cornea, including punctate epithelial keratopathy, filamentary keratitis, and indolent ulceration. Long-term use may also produce conjunctival edema and scarring, as well as follicular conjunctivitis, punctal edema and occlusion, ptosis, contact blepharodermatitis, and preauricular adenopathy. The routine use of topical antibiotics to manage herpes simplex infection is unwarranted and may add to drug toxicity. Corticosteroid therapy is contraindicated in the presence of active viral replication. Simple debridement may be as effective as antiviral therapy. Question 58 of 130 Which of the following systemic diseases may be associated with spontaneous subconjunctival hemmorhage?

Type II diabetes

Trisomy 21

Protein S deficiency

Hypertension

Please select an answer Feedback: Hypertension is known to be a risk factor for subconjunctival hemorrhage, as are anticoagulants or blood thinners. Protein S defeciency may lead to thrombosis. Type II diabetes and Trisomy 21 are not associated with spontaneous subconjunctival hemorrhage.

Question 59 of 130 Which of the following is not an adjunctive treatment of stromal ulceration after an alkali burn?

Acetylcysteine

Ascorbic acid

Flurbiprofen

Citrate

Please select an answer

Page 31: Cornea/External Disease

Feedback: Topical collagenase inhibitors such as L-cysteine, acetylcysteine, and disodium EDTA may be helpful in reducing collagenase activity, which leads to stromal melting. Topical and systemic ascorbic acid and citrate may significantly reduce the incidence of corneal ulceration and perforation. Topical nonsteroidal anti-inflammatory agents such as flurbiprofen have not been shown to be effective.

Question 60 of 130 Which statement about corneal injury from birth trauma is not true?

The breaks are usually horizontal.

The lesion may result in high astigmatism.

The patient may develop amblyopia.

Traumatic edema may clear initially.

Please select an answer Feedback: Ruptures in Descemet's membrane are usually vertical. Following birth trauma, the endothelium generally heals, leaving a hypertrophic ridge of Descemet's membrane. The corneal edema initially present may or may not clear. Even if it does clear, the cornea may decompensate later in life. Haab's striae in congenital glaucoma are typically horizontal or concentric to the limbus. Birth trauma should always be considered in the neonate with a cloudy cornea.

Question 61 of 130 The presence of multilayer endothelial cells that look and behave like epithelial cells is termed which one of the following?

Fuchs endothelial dystrophy

Posterior amorphous corneal dystrophy

Congential hereditary endothelial dystrophy

Posterior polymorphous dystrophy

Please select an answer Feedback: Posterior polymorphous dystrophy is an uncommon, slowly progressive autosomal dominant or autosoml recessive dystrophy that has a variable clinical spectrum. This dystrophy is characterized by the presence of multilayer endothelial cells that look and behave like epithelial cells. Clinically, the posterior corneal surface may demonstrate isolated group vesicles, geographic-shaped discrete gray lesions, and broad bands with scalloped edges. Stromal edema, corectopia, or broad iridocorneal adhesions can develop, and glaucoma can also occur. Severe disease can be managed with corneal transplantation. Fuchs endothelial dystrophy is a common condition that results from the primary dysfunction of the endothelial cells manifesting as corneal guttae, loss of endothelial cells, thickening of Descemet’s membrane, and eventually corneal edema. Posterior amorphous corneal dystrophy is a rare stromal dystrophy that manifests as diffuse grey-white sheetlike stromal opacities concentrated in the posterior stroma. Descemet’s membrane can be involved with focal areas of endothelial disruption. Congenital hereditary endothelial dystrophy (CHED) is due to a primary dysfunction of the corneal endothelial cells characterized by increased permeability and abnormal Descemet’s membrane secretion that manifests either at birth (CHED type 2, autosomal recessive), or during the first or second year of life (CHED type 1, autosomal dominant).

Page 32: Cornea/External Disease

Question 62 of 130 What is the protozoa that most commonly causes infectious keratitis in healthy contact lens wearers?

Herpes simplex

Acanthamoeba

Microsporidia

Pseudomonas

Please select an answer Feedback: Acanthamoeba is a ubiquitous free-living protozoa that is found in fresh water and soil. It is frequently associated with keratitis in contact lens wearers, although non-contact lens wearers with occupational exposure to fresh water and organic matter are also susceptible to the infection. Patients often present with extreme eye pain out of proportion to physical findings, and they are often misdiagnosed initially, resulting in a delay in appropriate treatment. This delay in treatment is associated with poorer outcomes. Microsporidia are intracellular protozoa that are typically found as an opportunistic infection in individuals infected with the human immunodeficiency virus. In these individuals, microsporidia presents as a diffuse conjunctivitis and epithelial keratitis. In immunocompetent individuals, microsporidia presents as a stromal keratitis. Pseudomonas keratitis is associated with contact lens wear, but it is not a protozoa. Herpes simplex virus (HSV) is also not protozoa, and there is no data to suggest that keratitis caused by HSV is more common in contact lens wearers.

Question 63 of 130 What is the most common causative agent for Parinaud oculoglandular syndrome?

Sporotrichosis

Bartonella henselae

Tularemia

Tuberculosis

Please select an answer Feedback: Parinaud oculoglandular syndrome is a granulomatous conjunctivitis with regional lymphadenopathy that is most commonly caused by Cat-scratch disease, which is primarily caused by Bartonella henselae. Other infrequent cases of Parinaud oculoglandular syndrome include tularemia, tuberculosis, and sporotrichosis. Clinically, a unilateral granulomatous conjunctivitis with one or more raised or flat, gelatinous, hyperemic, granulomatous lesions develops on the tarsal conjunctiva, fornix, or bulbar conjunctiva approximately 3-10 days after inoculation. Up to 2 weeks later, unilateral, firm and tender regional preauricular and submandibular lymph nodes develop. Mild systemic symptoms of fever, malaise, headache, and anorexia can also develop. Severe disseminated complications can also occur, but are uncommon. Treatment regimens vary in efficacy, but include a variety of antibiotics including azithromycin, erythromycin, and doxycycline.

Page 33: Cornea/External Disease

Question 64 of 130 The findings of vertigo and hearing loss are associated with which of the following causes of corneal stromal keratitis?

Varicella zoster virus interstitial keratitis

Syphilitic interstitial keratitis

Herpes simplex virus interstitial keratitis

Cogan syndrome

Please select an answer Feedback: Cogan syndrome is an autoimmune disease that produces the findings of stromal keratitis, vertigo, and hearing loss. The etiology of Cogan syndrome is obscure, but it typically affects young adults, often shortly after an upper respiratory infection. Early corneal findings consist of bilateral faint white subepithelial infiltrates in the periphery. Multifocal nodular infiltrates may develop later in the course of the disease in the posterior cornea. The acute keratitis of Cogan syndrome is treated with topical corticosteroids, but vestibular symptoms must be treated with systemic corticosteroids to improve the chances of preventing permanent hearing loss. Syphilitic keratitis can be congenital or acquired, but interstitial keratitis (IK) specifically is an immune-mediated manifestation of congenital syphilis. While it can be associated with cranial nerve VIII deafness, it is not generally associated with vertigo. Both herpes simplex virus IK and varicella zoster virus IK are not generally associated with vertigo or hearing loss.

Page 34: Cornea/External Disease

Question 65 of 130 One of the primary advantages of performing deep anterior lamellar keratoplasty over full thickness penetrating keratoplasty is the ability to avoid which type of rejection?

Epithelial

Endothelial

Stromal

Subepithelial

Please select an answer Feedback: Deep anterior lamellar keratoplasty has undergone a renewed interest recently, and the procedure involves removing the maximal amount of corneal stromal tissue while preserving the host Descemet’s membrane and endothelium. It is indicated for corneal diseases that spare Descemet’s memebrane and endothelium such as keratoconus or corneal scars that are not full-thickness. The procedure offers several advantages over traditional penetrating keratoplasty including the elimination of endothelial rejection since the host endothelium is preserved, a shorted healing time, and less stringent requirements for donor tissue. Question 66 of 130 When ocular surface squamous neoplasia penetrates through the underlying basement membrane, what is the correct term?

Severe conjunctival intraepithelial neoplasia (CIN)

Moderate conjunctival intraepithelial neoplasia (CIN)

Mild conjunctival intraepithelial neoplasia (CIN)

Squamous cell carcinoma

Please select an answer Feedback: In CIN, the neoplastic process does not invade the underlying basement membrane. As soon as it does, it is termed squamous cell carcinoma. On the ocular surface, squamous cell carcinoma appears as a plaque-like, gelatinous, or papilliform growth, usually at the limbus or on the bulbar conjunctiva, and typically in the interpalpebral fissure zone. Ultraviolet radiation is the main risk factor, but viral and genetic factors have also been identified to play a role. In HIV-positive individuals, the disease is more common and more aggressive. Treatment consists of excision, sometimes with adjuvant therapy with topical chemotherapeutic agents such as mitomycin.

Question 67 of 130 What is the syndrome with a central corneal opacity associated with variable degrees of iridocorneal adhesion extending from the iris collarette to the border of the opacity which is present from birth?

Sclerocornea

Cornea plana

Axenfeld-Rieger syndrome

Peter's anomaly

Please select an answer

Page 35: Cornea/External Disease

Feedback: Peter's anomaly is a central corneal opacity associated with variable degrees of iridocorneal adhesion extending from the iris collarette to the border of the opacity that is present from birth. Approximately 60% of cases are bilateral, and other ocular abnormalities are present in half of the cases. In the area of the opacity, Descemet membrane and endothelium are missing. In 60% of cases, systemic malformations such as developmental delay, heart defects, external ear abnormalities, hearing loss, CNS defects, and skeletal anomalies can be found. Axenfeld-Rieger syndrome is a spectrum of disorders characterized by posterior embryotoxon with attached iris strands, iris hypoplasia, and glaucoma with associated skeletal and dental abnormalities. Cornea plana refers to a flat cornea with a radius of curvature of less than 43 D. Corneal curvature that is the same as the adjacent sclera is pathognomonic. Sclerocornea also demonstrates a flat cornea, but there is also a loss of corneal transparency present.

Question 68 of 130 Which one of the following is an objective criteria for diagnosing Sjogren syndrome?

Schirmer II test of <5 mm in 5 minutes

Schirmer I test of <5 mm in 5 minutes

Positive fluorescein staining of the cornea and conjunctiva

Rose Bengal score of <4 Bijsterveld score

Please select an answer Feedback: While the precise cause of aqueous tear deficiency (ATD) dry eye in the setting of Sjogren syndrome (SS) is unknown, it is thought to be a T-cell mediated inflammatory disease resulting the destruction of lacrimal glands. Patients with ATD are considered to have SS if they have associated hypergammaglobulinemia, rheumatoid arthritis, and antinuclear antibody. In addition, SS is divided into 2 subets: primary and secondary. The revised International Classification Criteria for the diagnosis of SS defines 6 criteria: ocular symptoms, oral symptoms, ocular signs, histopathologic features, salivary gland involvement, and autoantibodies. Primary SS is diagnosed with the presence of 4 out of the 6 criteria, or the presence of 3 of the last 4 criteria (objective criteria). Secondary SS is diagnosed with the combination of a positive response to one of the first 2 criteria plus a positive response to at least 2 items from ocular signs, histopathologic features, or salivary gland involvement. A positive response to ocular signs is objective evidence of ocular involvement, determined on the basis of a positive result on at least 1 of the following 2 tests: Schirmer I test of <5mm in 5 minutes or Rose Bengal score of > 4 Bijsterveld score.

Question 69 of 130 Infection with Chlamydia trachomatis serotypes A-C results in which one of the following?

Lymphogranuloma venereum

Adult inclusion conjunctivitis

Neonatal inclusion conjunctivitis

Trachoma

Please select an answer Feedback: Trachoma is caused by infection with Chlamydia trachomatis serotypes A-C whereas adult and neonatal inclusion conjunctivitis are caused by Chlamydia trachomatis serotypes D-K, and lymphogranuloma venereum is caused by Chlamydia trachomatis serotypes L1-L3. Trachoma affects approximately 150 million individuals worldwide, and it is the leading cause of preventable blindness. This infection occurs in communities with poor hygiene and inadequate sanitation, and transmission is usually eye-to-eye or by flies and other household fomites. The diagnosis of trachoma is made by observing 2 of the following 4 findings: conjunctival follicles on the upper tarsal conjunctiva, limbal follicles and their sequela (Herbert’s pits), typical tarsal conjunctival scarring, and vascular pannus most marked on the superior limbus. Treatment of active trachoma consists of topical and oral tetracycline, erythromycin, or azithromycin.

Page 36: Cornea/External Disease

Question 70 of 130 Which medication can cause deposits in the corneal stroma?

Gold

Chloroquine

Amiodarone

Indomethacin

Please select an answer Feedback: The use of gold has been associated with a condition called chrysiasis, which manifests as gold deposits in the deep stroma, usually in the periphery. Amiodarone, chloroquine, and indomethacin all deposit in the epithelial layer of the cornea, manifesting in a whorl-like pattern termed cornea verticillata. Typically, these deposits do not affect vision.

Question 71 of 130 Failure to treat gonococcal conjunctivitis may lead to which of the following conditions?

Necrotizing scleritis

Orbitall cellulitis

Corneal ulceration and perforation

Chancre formation

Please select an answer Feedback: Either untreated or inadequately treated cases of gonococcal conjunctivitis can progress rapidly to corneal ulceration and perforation. This progression often begins with a peripheral corneal infiltration and subsequent liquefaction.

Question 72 of 130 What is the most common cause of infectious crystalline keratopathy?

Pseudomonas species

Alpha-Hemolytic Streptococcus species

Nocardia species

Staphylococcous species

Please select an answer Feedback: Infectious crystalline keratopathy (ICK) occurs when a colony of slow-growing organisms become sequestered within the corneal stroma. Premissive compromise of the corneal inflammatory response is usually necessary. The classical description of ICK is the presence of densely packed, white, branched aggregates of organisms without significant keratitis. Coritcosteroid use, contact lens wear, and corneal grafts can all predispose to this condition. Although reported to be caused by a number of different organisms, the most common culprit is Alpha-Hemolytic Streptococcus species.

Page 37: Cornea/External Disease

Question 73 of 130 What is the most appropriate therapeutic regimen for a patient with adult inclusion disease?

Ceftriaxone 1g intramuscularly

Topical sodium sulfacetamide 10% 4 times daily

Aqueous penicillin G, 10 million units intramuscularly

Oral tetracycline 1 to 1.5g per day plus topical 1% tetracycline 4 times daily for 3 weeks

Please select an answer Feedback: In adult inclusion disease, a combined regimen of oral and topical tetracycline is recommended for 3 weeks. Alternatives to this regimen in the event of intolerance or contraindication to tetracycline include oral erythromycin 1g per day for 3 weeks or doxycycline (loading dose of 200mg followed by 100mg twice daily for 3 weeks) and topical 0.5% erythromycin ointment 4 times daily. Tetracycline is contraindicated in pregnancy because of its tendency to lead to staining of the teeth in childhood. Initiation of systemic therapy for adult chlamydial disease in a pregnant woman should be preceded by consultation with the patient's obstetrician.

Question 74 of 130 A 22-year-old man complains of tearing, photophobia, and the appearance of a white spot in the center of his cornea for 1 day. He wore contact lenses until the onset of his symptoms. When examined, he is noted to have a markedly thickened central cornea with opacification. In the other eye, the cornea is thinned centrally and there is slight apical scarring. Which of the following is not an appropriate treatment for this patient?

Surgery

Hypertonics

Cycloplegia

Patching

Please select an answer Feedback: Hydrops usually heals spontaneously over a period of several weeks, although when the edema resolves, there is often residual scarring. Some patients have good vision after the corneal hydrops has healed. Therefore, the condition is treated conservatively with topical hypertonic agents, patching (or alternatively a soft contact lens), and, occasionally, cycloplegia for "ciliary pain." The scarring that occurs after hydrops may actually flatten the cornea and give the patient some improvement in vision.

Page 38: Cornea/External Disease

Question 75 of 130 A 21-year-old man has had a red, irritated right eye for 4 weeks. He states that he was seen initially in an outpatient emergency clinic after 1 week of symptoms and was given antibiotic drops, which he took 4 times a day for 10 days. Despite using the eyedrops, he has noticed an increase in the discharge from his right eye, and in the past 2 days his left eye has become red and irritated and has developed a yellow mucoid discharge. He complains of only mild blurring of vision in the right eye. Examination reveals a coarse punctate epithelial keratitis and bilateral follicular conjunctivitis. The time course of the development of symptoms in this patient is most consistent with which of the following?

Pharyngoconjunctival fever

Hyperacute bacterial conjunctivitis

Adenoviral epidemic keratoconjunctivitis

Adult inclusion disease

Please select an answer Feedback: The symptoms of adenoviral conjunctivitis generally peak within the first week after onset, as the patient develops first diffuse and then focal epithelial keratitis between 3 and 5 days after the onset of the inflammatory response. In addition, the inflammatory response in epidemic keratoconjunctivitis may be severe enough to produce a membranous or pseudomembranous response rarely seen in adult inclusion disease. The patient with viral conjunctivitis, therefore, usually seeks medical attention early in the course of the disease. The ocular findings in adult inclusion disease appear 1 to 2 weeks after exposure to infectious genital secretions. The onset of these symptoms is not usually as acute as that of adenoviral keratoconjunctivitis, although it may be indistinguishable. Signs include a hyperemic papillary conjunctivitis that evolves into follicular conjunctivitis, a relatively scant mucopurulent discharge, and palpable preauricular adenopathy. The presence of limbal follicles distinguishes chlamydial conjunctivitis from viral infection. The keratitis of adult inclusion disease, which is characterized by punctate keratopathy most commonly found in the superior cornea as well as small, focal subepithelial infiltrates, does not appear until the second week of the disease and is accompanied by an exacerbation of the symptoms. Question 76 of 130 Which of the following is not an acceptable intraocular lens implant fixation techniques used during an implant exchange in the surgical management of pseudophakic bullous keratopathy?

Ciliary sulcus placement of an all-PMMA posterior chamber IOL

Anterior chamber placement of a 4-point (Kelman-style) IOL

Iris fixation of a posterior chamber IOL with a PMMA optic and polypropylene haptics

Ciliary sulcus placement of a silicone plate-style posterior chamber IOL

Please select an answer Feedback: In a patient without adequate capsular support for a posterior chamber IOL, a 4-point fixation-style (Kelman-style) IOL is another option. This technique has the advantage of decreasing the time that the eye is open during penetrating keratoplasty when compared to iris or ciliary sulcus fixation of the IOL. Plate-style posterior chamber silicone implants require an intact capsular bag for stabilization.

Page 39: Cornea/External Disease

Question 77 of 130 A 21-year-old man has had a red, irritated right eye for 4 weeks. He states that he was seen initially in an outpatient emergency clinic after 1 week of symptoms and was given antibiotic drops, which he took 4 times a day for 10 days. Despite using the eyedrops, he has noticed an increase in the discharge from his right eye, and in the past 2 days his left eye has become red and irritated and has developed a yellow mucoid discharge. He complains of only mild blurring of vision in the right eye. Examination reveals a coarse punctate epithelial keratitis and bilateral follicular conjunctivitis. Further inspection of the external eye of the patient reveals the presence of follicles on the superior tarsal conjunctiva. Which diagnostic test is least useful in the early diagnosis of chlamydial infection?

Conjunctival scraping for fluorescein-tagged antibodies

Conjunctival scraping and staining with Giemsa stain

Systemic antibody titers

Inoculation of cell cultures

Please select an answer Feedback: Systemic antibody titers are not generally useful in the diagnosis of acute chlamydial infection. Examination of a Giemsa-stained conjunctival scraping to determine specific cell type and cytopathic effects may be used to establish a diagnosis. The diagnosis can be confirmed rapidly by use of fluorescein-tagged direct or indirect antibody staining for chlamydial antigen on a conjunctival scraping that is air-dried on a glass slide. Confirmation of immunofluorescent staining can be achieved by using cell cultures inoculated with scrapings of the conjunctiva. Another diagnostic alternative is the use of an enzyme-linked immunosorbent assay (ELISA).

Question 78 of 130 Irregular astigmatism is most commonly caused by which of the following dystrophies?

Fleck dystrophy

Epithelial basement membrane dystrophy

Macular corneal dystrophy

Central cloudy dystrophy of Francois

Please select an answer Feedback: Corneal epithelial basement membrane dystrophy is the most common anterior corneal dystrophy, and can lead to decreased vision from induced irregular astigmatism. Irregular astigmatism results from irregular corneal contour from unevenly thickened basement membrane, extension of deposits between epithelial layers and deposition of fibrillar material between the corneal basement membrane and Bowman layer. Macular corneal, fleck, and central cloudy dystrophy of Francois primarily affect the corneal stroma, which typically does not result in irregular astigmatism.

Page 40: Cornea/External Disease

Question 79 of 130 Which of the following statements about untreated adult inclusion disease is true?

The patient often develops an associated polyarthritis.

The natural duration of the disease is 6 to 18 months.

The disorder rarely lasts longer than 2 months.

Chronic infection causes symblepharon and mucosal destruction.

Please select an answer Feedback: Persistent untreated adult inclusion conjunctivitis does not represent a vision-threatening problem but often lasts for many months, during which time it can be transmitted. It is generally not associated with severe conjunctival scarring or joint symptoms.

Question 80 of 130 A 21-year-old man has had a red, irritated right eye for 4 weeks. He states that he was seen initially in an outpatient emergency clinic after 1 week of symptoms and was given antibiotic drops, which he took 4 times a day for 10 days. Despite using the eyedrops, he has noticed an increase in the discharge from his right eye, and in the past 2 days his left eye has become red and irritated and has developed a yellow mucoid discharge. He complains of only mild blurring of vision in the right eye. Examination reveals a coarse punctate epithelial keratitis and bilateral follicular conjunctivitis. Which of the following events in the recent history of this patient is least likely to be of central importance in determining the diagnosis?

Lower-back pain

Exposure to a family member with conjunctivitis

Upper respiratory tract infection

Urethritis

Please select an answer Feedback: A history of contact that might be the source of fomitic spread is important. Adult inclusion conjunctivitis is a venereally transmitted disease usually found in conjunction with nonspecific urethritis or cervicitis. It is therefore important to ask the patient about symptoms of genital discharge and of similar symptoms in a sexual partner. Acute follicular conjunctivitis resulting from adenoviral infection may often be associated with a contiguous upper respiratory tract infection or with exposure to a family member or friend who has conjunctivitis.

Question 81 of 130

Which of the following ingredients within topical ocular anti-hypertensive agents are most toxic?

Preservatives

Vehicle

Buffer

Active ingredient

Please select an answer

Page 41: Cornea/External Disease

Feedback: The "preservative," or antiseptic chemical(s) within most eye drops is the most toxic ingredient in the bottle. The corneal epithelium and conjunctiva may serve as depots for these agents. Benzalkonium chloride is among the most extensively studied preservative. Residual preservatives are detectable in the corneal epithelium for days following a single topical application. Corneal application of these preservatives can cause morphologic epithelial changes which include loss of microvillae, plasma membrane disruption, and cellular loss.

Question 82 of 130

A pedunculated papilloma of the conjunctiva is caused by which virus?

Human papilloma virus (HPV)

Human immunodeficiency virus (HIV)

Herpes simplex virus (HSV)

Influenza virus

Please select an answer Feedback: HPV subtypes 6 and 11 initiate a neoplastic growth of epithelial cells with vascular proliferation that gives rise to a verruca vulgaris of the eyelid skin or a pedunctulated papilloma of the conjunctiva. Although these lesions can be more extensive in individuals with HIV infection, it is not caused by the HIV virus itself.

Question 83 of 130 Which of the following medications is associated with vortex keratopathy?

Gold

Retinoids

Hydroxychloroquine

Phenothiazines

Please select an answer Feedback: Corneal verticillata is the term for the whorl-like deposition pattern in the corneal basal epithelium of the cornea. Verticillata resembles the pattern seen in Fabry disease. This pattern of deposits can be seen with many systemic medications including hydroxychloroquine, amiodarone, chlorpromazine, indomethacin, naproxen, and tamoxifen. Discontinuing medications is not usually indicated, because corneal deposits rarely cause visual disturbances and usually disappear following discontinuation. Gold, phenothiazines, and retinoids are known to cause corneal stromal deposits.

Page 42: Cornea/External Disease

Question 84 of 130 A patient has a central corneal scar that extends into the mid-stroma, but Decements membrane is unaffected. Which of the following surgical procedures has the highest likelihood to rapidly improve visual function?

Lamellar keratoplasty

LASIK surgery

Phototherapeutic keratectomy

Penetrating keratoplasty

Please select an answer Feedback: For a central corneal scar that extends into the mid-stroma, neither phototherapeutic keratectomy or superficial keratectomy will remove the opacified and involved deeper stromal scar. LASIK is not indicated in this situation, as opacification would be present in the cap. Although a penetrating keratoplasty would remove the scar, a lamellar keratoplasty offers a less invasive procedure, quicker recovery, earlier wound strength, and no risk of endothelial rejection.

Question 85 of 130 Splash injury from strongly alkaline chemicals leads to which chemical change contributing to tissue damage?

Saponification of fatty acids

Lengthening of stromal collagen

Decreased aqueous pH

Covalent cross-linking of proteoglycans

Please select an answer Feedback: Following chemical burn of the cornea, strong alkalis cause saponification of fatty acids in cell membranes that leads to cellular dissolution. Once the surface epithelium is lost, alkaline solutions penetrate readily through the corneal stroma, where the proteoglycan ground substance and collagen fibers are rapidly destroyed. Strong alkalis also raise the pH of tissues, and if they penetrate the anterior chamber, they can damage uveal and angle tissues, and induce intense inflammation.

Question 86 of 130

A healed corneal laceration may cause visual loss from corneal opacification, or regular or irregular astigmatis. Which management treats the irregular astigmatism most cost-effectively?

Astigmatic keratectomies

Penetrating keratoplasty

Excimer laser treatment

Rigid gas-permeable contact-lens fitting

Please select an answer Feedback: Rigid gas-permeable contact-lens fitting is a low cost and highly effective initial management for irregular astigmatism resulting from healed corneal lacerations. In addition, a rigid contact lens refraction provides a measure of

Page 43: Cornea/External Disease

visual potential for evaluating the risk-reward ratio if future surgical intervention is considered. Frequently, vision can be improved with a rigid contact lens to the point where penetrating keratoplasty is not needed. If regular astigmatism predominates over irregular astigmatism, surgical alternatives can be considered, such as lamellar keratoplasty and astigmatic keratectomies.

Page 44: Cornea/External Disease

Question 87 of 130

By which mechanism does Pseudomonas aeruginosa adhere to the cornea?

Adheres to surfactants in the protien tear layer

Binds to receptors on injured epithelial cells

Extending pseudopodia into basal epithelial layer

Binds to collagen of the exposed Bowman layer

Please select an answer

Feedback: Psudomonas aeruginosa adheres to the cornea by binding to surface receptors that become exposed on

injured epithelial cells. Within hours, a clone of bacteria invades the cornea between stromal lamellae. Staphylococcus

aureaus adheres to collagen and other components of the exposed Bowman layer and stroma. Surfactants and

pseudopodial extension are not known to be involved in initial bacterial adhesion. aeruginosa adheres to the cornea by

binding to surface receptors that become exposed on injured epithelial cells. Within hours, a clone of bacteria invades the

cornea between stromal lamellae. Staphylococcus aureaus adheres to collagen and other components of the exposed

Bowman layer and stroma. Surfactants and pseudopodial extension are not known to be involved in initial bacterial

adhesion.

Question 88 of 130

Which corneal degeneration is characterized by a painless ectasia of the cornea along an inferior band?

Terrien's marginal degeneration

Pellucid marginal degeneration

Keratoglobus

Keratoconus

Please select an answer Feedback: Pellucid marginal degeneration is one of the non-inflammatory ectatic degenerations that is characterized by protrusion of the cornea in the region of an inferior corneal band of thinning. No vascularization or lipid deposition occurs. In contrast, keratoconus is characterized by progressive thinning of the central cornea. Keratoglobus is characterized by global thinning of all of the cornea. Terrien's marginal degeneration is characterized by marginal thinning with vascularization and lipid deposition.

Page 45: Cornea/External Disease

Question 89 of 130

When repairing a full-thickness corneal laceration, the surgeon should attempt to optimally reconstitute corneal curvature. Compared to the peripheral corneal sutures, central corneal suture placement should follow which preference?

Widely spaced and longer

Widely spaced and shorter

Closely spaced and longer

Closely spaced and shorter

Please select an answer Feedback: To reconstitute corneal curvature, the repair of a full-thickness corneal laceration should utilize non-uniform suture placement and length. To flatten the peripheral cornea, widely spaced, longer sutures should be used to close the laceration in the periphery. To steepen the central cornea, closely spaced, shorter sutures should be used to close the central portions of the laceration. Suture placement across the visual axis, which could induce irregular astigmatism, scarring or excessive flattening should be avoided if possible.

Question 90 of 130

In contrast to nonfilamentous keratitis (such as from Candida species), filamentary fungal keratitis is more often associated with which risk factor?

Chronic bandage contact-lens wear

Keratoconjunctivitis sicca

Corneal trauma

Cool and temperate climates

Please select an answer

Feedback: Fungal keratitis caused by filamentous fungi most often follows ocular trauma with vegetative matter. The

prevalence of filamentous increases in warm, humid environments. Filamentous fungal infections are not typically

associated with dry eyes or chronic bandage contact-lens wear.

Page 46: Cornea/External Disease

Question 91 of 130 Which layer of the cornea is most susceptible to damage from ultraviolet radiation?

Bowman membrane

Endothelial cells

Epithelial cells

Stroma

Please select an answer Feedback: The corneal epithelium is highly suceptible to injury from ultraviolet radiation. The symptoms of pain and decreased vision can occur up to a few hours after exposure, but are generally self-limited.

Question 92 of 130

Superior limbic keratoconjunctivitis can be associated with which of the following systemic conditions?

Sarcoidosis

Thyroid eye disease

Systemic Lupus Erythematosis

Rheumatoid arthiritis

Please select an answer Feedback: Superior limbic keratoconjunctivitis (SLK) is characterized by a fine papillary reaction of the superior tarsal conjunctiva, injection and thickening of the superior bulbar conjunctiva, hypertrophy of the superior limbus, fine punctate staining of the superior bulbar conjunctiva, and superior corneal filamentary keratitis. It has been associated with autoimmune thyroid disease, but not sarcoidosis, lupus, nor rheumatoid arthritis.

Question 93 of 130 A break in which structure leads to corneal hydrops associated with keratoconus?

Bruch's membrane

Epithelial basement membrane

Bowman's layer

Descemet's membrane

Please select an answer Feedback: Corneal hydrops associated with keratoconus is caused by a tear in Descemet's membrane. Breach of Descement's membrane disrupts the endothelial continuity followed by rapid corenal edema, decreased vision, and pain. Although breaks in Bowman's layer are seen in keratoconus, this injury does not result in hydrops. Bruch's membrane lies between the retina and retinal pigmented epithelium and is not affected by keratoconus.

Page 47: Cornea/External Disease

Question 94 of 130

Which is the best initial management to achieve visual rehabilitation for a isolated peripheral corneal scar?

Rotational autograft

Limbal relaxing incisions

Hard contact lens

Phototherapeutic keratectomy

Please select an answer Feedback: Peripheral corneal scars that do not involve the entrance pupil can cause visual loss by inducing irregular astigmatism. Prior to any surgical procedures, a hard contact lens fitting should be performed because this can usually correct the vision. Occasionally, limbal relaxing incisions can help decrease the amount of astigmatism, but the results are unpredicatable. In addition, phototherapeutic keratectomy is not indicated, nor is a rotational autograft because this procedure is used to rotate a scar out of the entrance pupil.

Question 95 of 130

In Wilson disease, copper is deposited in which layer of the cornea?

Stroma

Bowman layer

Epithelium

Descemet membrane

Please select an answer Feedback: Wilson disease, or hepatolenticular degeneration, is an autosomal recessive metabolic defect where copper is deposited first in the liver, then kidneys, and eventually the brain and Descemet membrane of the cornea, specifically in the posterior lamella of Descemet membrane. It appears as a golden brown, ruby red, or green pigment (Kayser-Fleischer ring) in the peripheral Descemet membrane, starting superiorly, gradually spreading and widening to meet deposits inferiorly. There is no visible accumulation of copper in the epithelium, Bowman layer or stroma. Question 96 of 130 Which one of the following dystrophies has the lowest chance of recurrence in the graft after keratoplasty?

Lattice corneal dystrophy

Macular corneal dystrophy

Avellino dystrophy

Granular corneal dystrophy

Please select an answer Feedback: Classically, it has been thought that lattice dystrophy had the highest recurrence rate after corneal grafting, but recent data from one study suggests that granular dystrophy may have a higher rate. Avellino dystrophy is a combination of lattice and granular dystrophy. Macular dystrophy has the lowest rate of recurrence after keratoplasty.

Page 48: Cornea/External Disease

Question 97 of 130 What is the preferred initial therapy when treating peripheral, painful, post-traumatic, recurrent corneal erosions.

Penetrating keratoplasty

Anterior stromal puncture

Lamellar keratoplasty

Excimer laser phototherapeutic keratectomy

Please select an answer Feedback: The initial managment of corneal erosions is conservative therapy with topical lubricants, hypertonic saline solutions, and bandage contact lenses. This option is not given. For recurrent peripheral corneal erosions the subsequent management depends on the etiology of the erosions. For eyes which have post-traumatic recurrent erosions which do not involve the axial cornea (affecting the entrance pupil) anterior stromal micropuncture can be very effective. This procedure produces a firm adhesion between the epithelium and the underlying stroma. Because it can cause visually significant scarring, this procedure should be used with caution in central or axial cornea. In patients with dystrophic, degenerative, or other secondary basement membrane disorders causing recurrent erosions, epithelial debridement is generally the preferred procedure. Excimer laser phototherapeutic keratectomy is an effective modality, but is generally tried after other procedures have been attempted. Lamellar or penetrating keratoplasty is not indicated for recurrent corneal erosions.

Question 98 of 130 You examine a patient 3 months after successful surgical repair of a full-thickness corneal laceration. Which of the following findings would be most likely at the site of injury?

Irregularity in Bowman's layer

Increased thickness of collagen fibrils

Gap in Descemet's membrane

Absence of endothelial cells

Please select an answer Feedback: As in most other tissues, the cornea heals by fibrosis rather than fibrovascular proliferation. Three months after a penetrating injury, the cornea would be expected to be resurfaced by healed corneal epithelium, have endothelial cells that have extended and/or migrated to cover the defect, and secreated additional Descemet membrane from the endothelial cells. The fibroblasts of the stroma will have become activated, migrating across the wound and laying down collagen and fibronectin. However, Bowman's layer (which is not a true membrane, but a compacted collagen of the anterior stroma) does not regenerate when incised or destroyed, and thus an irregularity or discontinuity will be present.

Question 99 of 130 What finding following strabismus surgery suggests the diagnosis of anterior segment ischemia?

Breaks in Bowman's layer

Breaks in Descemet's membrane

Corneal perforation

Folds in Descemet's membrane

Please select an answer Feedback: Anterior segment ischemia can occur after strabismus surgery. This develops from transection of several anterior ciliary arteries which enter the eyewall at the rectus muscle insertions. The earliest finding of anterior segment ischemia are cells and flare in the anterior chamber. More severe cases are characterized by corneal epithelial edema, folds

Page 49: Cornea/External Disease

in Descemet's membrane, and other signs of anterior uveitis. Severe cases can lead to anterior segment necrosis and phthisis bulbi. Breaks in Descemet's membrane and Bowman's layer are seen in keratoconus.

Question 100 of 130

Following an alkali burn to the cornea, you prescribe antimicrobial and/or anti-inflammatory therapy. During what period can you safely use intensive topical steroids?

Three days after topical antibiotic initiation

Intensive topical steroids are contraindicated following alkali burns

Over the first two weeks

Seven days after topical antibiotic initiation

Please select an answer

Feedback: After the initial acute management of a chemical burn (including irrigation and acute intraocular pressure

control), attention should be directed at decreasing inflammation, monitoring the intraocular pressure, limiting matrix

degradation, and promoting re-epithelialization of the cornea. Because an intense polymorphonuclear (PMN) leukocyte

infiltration of the corneal stroma occurs and may be a major source of proteolytic enzymes that can dissolve the corneal

stroma, corticosteroids should be used intensively during the first 2 weeks to inhibit PMN function. This dosage should be

decreased significantly after the first 2 weeks because steroids can inhibit wound healing and increase the risk for infection.

Question 101 of 130 Allergic contact blepharoconjunctivitis is mediated by what immune mechanism?

Type IV (delayed hypersensitivity)

Type I (anaphylactic)

Type II (cytotoxic hypersensitivity)

Type III (immune-complex reaction)

Please select an answer Feedback: Contact blepharoconjunctivitis is mediated by delayed Type IV hypersensitivity reaction. Allergic responses to medications usually begin 24-72 hours following instillation of a topical drop, cutaneous exposure or oral intake.. Type I reactions include anaphylactic and atopic reactions. Type II reactions are cytotoxic hypersensitivity reactions that include ocular cicatricial pemphigoid. Type II reactions are immune-complex reactions such as scleritis.

Page 50: Cornea/External Disease

Question 102 of 130 What finding after alkali chemical injury to the eye is the strongest risk factor for a poor prognosis?

Blanching of most of the perilimbal vessels

Total loss of corneal epithelium

360 degrees of limbal injection

corneal edema

Please select an answer Feedback: The most unfavorable visual prognosis is associated with extensive limbal stem-cell damage and intraocular chemical penetration. The limbus contains corneal epithelial stem cells, so damage to this area can lead to disruption in repopulating the corneal epithelium. Severe damage to the limbal area is seen as limbal blanching and represents ischemia and death of the cells.

Question 103 of 130 In a patient with chronic conjunctivitis, a conjunctival biopsy contained granulomas. The differential would include what condition?

Ocular cicatricial pemphigoid

Atopic conjunctivitis

Masquerade syndrome

Cat-scratch disease

Please select an answer Feedback: Parinaud oculoglandular syndrome can be caused by cat-scratch disease, with the most common causative agent being Bartonella henselae. This condition manifests as a unilateral granulomatous conjunctivitis with 1 or more raised or flat, gelatinous, hyperemic, granulomatous lesions on the superior or inferior palpebral conjunctiva, fornix, or bulbar conjunctiva. The other entities do not manifest as granulomatous lesions.

Question 104 of 130 What condition, which may be revealed by medical history, affects the management of patients initiating treatment for cicatricial pemphigoid/mucous membrane pemphigoid?

Glucose-6-phosphate dehydrogenase (G6PD) deficiency

Prematured birth

Cardiac anomalies

Asthma

Please select an answer Feedback: Dapsone is a drug that has been advocated by many ophthalmologists as the initial drug of choice for treating mild cases of ocular cicatricial pemphigoid/mucous membrane pemphigoid. However, it must be used cautiously in patients with G6PD or sulfa allergy because these patients can develop hemolytic anemia. Of importance, others may rarely develop hemolytic anemia. However, asthma, cardiac abnormalities and premature birth are not risk factors for hemolytic anemia.

Page 51: Cornea/External Disease

Question 105 of 130 Varicella zoster can cause chronic corneal inflammation which results in what permanent vision-reducing condition?

Punctate epithelial keratitis

Dendritic epithelial keratitis

Elevated mucous plaques

Lipid keratopathy

Please select an answer Feedback: Chronic corneal stromal inflammation can lead to corneal vascularization, lipid keratopathy, and corneal opacity. Punctate and dendritic epithelial keratitis are acute findings caused by viral replication in corneal epithelium. Elevated dendritiform mucous plaques may occur weeks to months after resolution of skin lesions, but do not lead to permanent visual compromise.

Question 106 of 130 A 25-year-old African-American patient with a history of sickle cell disease presents with an acute traumatic hyphema OS with an associated intraocular pressure of 38 mm Hg. What medication would be a relative contraindication in this setting?

Timolol maleate

Dorzolamide hydrochloride

Brimonidine tartrate

Atropine 1%

Please select an answer Feedback: In cases of traumatic hyphemas, sickle cell patients are predisposed to sickling of red blood cells in the anterior chamber, which can restrict their outflow through the trabecular meshwork and raise intraocular pressure dramatically. Carbonic anhydrase inhibitors such as dorzolamide (as well as osmotic agents) must be used cautiously with these patients because these agents tend to reduce pH and lead to hemoconcentration, both of which may exacerbate sickling of red blood cells.

Question 107 of 130 Following a traumatic hyphema, when is rebleeding most likely to occur?

0 to 1 days

2 to 5 days

11 to 14 days

6 to 10 days

Please select an answer Feedback: Rebleeding after a traumatic hyphema occurs in 3-30% of cases, and most frequently 2-5 days after injury. The timing of the rebleeding may be related to the lysis and clot retraction that occur during this period. Rebleeding is a prognostic factor for poor visual outcome, and approximately 50% of patients with rebleeding develop elevated intraocular pressure.

Page 52: Cornea/External Disease

Question 108 of 130 What condition can cause a chronic follicular conjunctivitis?

Chronic dacryocystitis

Bacterial blepharoconjunctivitis

Molluscum contagiosum

Keratoconjunctivitis sicca

Please select an answer Feedback:

Molluscum contagiosum eyelid lesions release viral particals into the tear film, causing chronic follicular conjunctivitis. Dry eyes and bacterial conjunctivits, whether from exogenous or lacrimal source, cause a papillary reaction of the palpebral conjunctiva.

Question 109 of 130 What area of the conjunctiva would be most likely to have prominent staining with rose bengal in a case of conjunctival medicamentosa?

Inferior palpebral

Superior limbal

Temporal bulbar

Inferior bulbar

Please select an answer Feedback: Toxic keratoconjunctivitis from medications is commonly encountered but frequently unrecognized. Often the toxicity is from the preservatives in the eyedrops, most notoriously benzalkonium chloride. In the mildest form, punctate epithelial erosions of the inferior cornea and bulbar conjunctiva can be seen. In more severe cases, a diffuse punctate epithliopathy with whorling can be seen, along with a papillary or follicular response of the palpebral conjunctiva. Corneal neovascularization from limbal stem cell damage can also occur.

Page 53: Cornea/External Disease

Question 110 of 130

Oral tetracyclines are given following an alkali burn to inhibit what process?

Infection

Collagenolysis

Endothelial cell dysfunction

Epithelialization

Please select an answer

Feedback: After a chemical burn to the cornea, an intense polymorphonuclear (PMN) leukocyte infiltration of the corneal

stroma occurs, and these PMNs may be a major source of proteolytic enzymes that are capable of dissolving corneal

stroma. Intensive topical corticosteroids are used in the first 2 weeks to inhibit PMN function. In addition, as oral

tetracyclines are potent chelators of extracellular calcium, and a deficiency of calcium in the plasma membrane of PMNs

inhibits their activity to degranulate, oral tetracyclines are used to inhibit collagenolysis. Oral tetracyclines do not

significantly alter extra-limbal bacteria, epithelialization or endothelial function.

Question 111 of 130 What is the traumatic setting in which cyanoacrylate tissue adhesive may be useful?

Small (< 3 mm) corneal perforation

Full-thickness scleral laceration

Full-thickness corneal laceration with uveal prolapse

Large (> 4 mm) corneal perforation

Please select an answer Feedback: Cyanoacrylate glue can be used very successfully to close small corneal perforations less than 3 mm in size. Larger perforations may be closed with sutures or in more difficult cases require tectonic corneal grafting. Corneal and scleral lacerations are generally amenable to conventional primary suturing techniques.

Question 112 of 130 Three months after the onset of herpes zoster ophthalmicus, a patient developed decreased vision and photophobia in the ipsilateral eye. Examination reveals an intact epithelium, moderate stromal edema, stromal vascularization, and keratic precipitates. What would be the preferred initial topical management?

Vidarabine

Prednisolone acetate

Erythromycin

Trifluridine

Page 54: Cornea/External Disease

Please select an answer Feedback: This scenario describes varicella zoster keratouveitis, which may be effectively treated with topical corticosteriods and cycloplegics. Topical trifluridine or vidarabine have no role in treating varicella zoster infections, nor does any antibiotic medication, except to prevent bacterial superinfections.

Question 113 of 130

What is the most common post-operative complication following excision of a pterygium?

Dehiscence of conjunctival graft

Recurrence

Persistent corneal epithelial defect

Scleral melting

Please select an answer Feedback: The most common post-operative complication of excision of a pterygium is late recurrence. The recurrence rate often depends on the excision technique chosen. Bare sclera techniques have a high recurrence rate, ranging from 40-75% whereas conjunctival autografting has a recurrence rate of 3-5%.

Question 114 of 130 What is the most common cause of chronic angular blepharitis?

Pseudomonas aeruginosa

Staphylococcus aureus

Moraxella lacunata

Streptococcus viridans

Please select an answer Feedback: Chronic angular blepharitis is most often caused by Moraxella lacunata, although it is frequently associated with Staphylococcus aureus belpharitis. Streptococcus viridans is often implicated in cases of infectious crystalline keratopathy, and pseudomonas aeruginosa is the common culprit in contact lens associated bacterial keratitis.

Question 115 of 130

What structural layer within the cornea is related to a Kayser-Fleisher ring?

Descemet membrane

Epithelium

Stroma

Bowman layer

Page 55: Cornea/External Disease

Please select an answer Feedback: A Kayser-Fleischer ring is a result of copper deposition in the posterior lamella of the Descemet membrane. The ring is seen as a golden brown, ruby red, or green pigment ring in the peripheral Descemet membrane. This finding is most frequently found in individuals with Wilson disease.

Question 116 of 130 What corneal finding is associated with Cogan syndrome?

Endothelitis

Keratouveitis

Epithelial keratitis

Stromal keratitis

Please select an answer Feedback: Cogan syndrome is an autoimmune disorder that produces stromal keratitis, vertigo, and hearing loss. The etiology of the syndrome is unknown, but it shares features with polyarteritis nodosa. The earliest corneal findings include bilateral, faint, white, subepithelial infiltrates in the peripheral cornea. Later, multifocal nodular infiltrates may develop in the posterior cornea.

Question 117 of 130 Used following alkali corneal burns, what therapy promotes collagen synthesis?

Citric acid

Medroxyprogesterone

Ascorbic acid

Tetracycline

Please select an answer Feedback: In the setting of alkali corneal burns, high-dose ascorbic acid is thought to promote collages synthesis, whereas oral tetracyclines and citric acid inhibit polymorphonuclear cell-induced collagenolysis, and medroxyprogesterone suppresses collagen breakdown.

Question 118 of 130

Allergic conjunctivitis represents what type of hypersensitivity response?

Type III

Type I

Type II

Type IV

Please select an answer Feedback: In Type I reaction, such as in allergic conjunctivitis, antigens combine with IgE antibodies bound to receptors on mast cells to cause a release of histamine and other mediators as well as new synthesis of prostaglandins and leukotrienes.

Page 56: Cornea/External Disease

Ocular cicatricial pemphigoid is an example of Type II reaction, scleritis is an example of a Type II reaction, and phlyctenulosis is an example of Type IV reaction.

Question 119 of 130 What corneal dystrophy is associated with recurrent corneal erosions?

Lattice corneal dystrophy

Posterior polymorphous dystrophy

Central cloudy dystrophy of Francois

Fleck dystrophy

Please select an answer Feedback: Among the common corneal stromal dystrophies (granular, lattice, and macular) recurrent corneal erosions can be seen most frequently in lattice dystrophy. Erosions are not seen in posterior polymorphous dystrophy, fleck dystrophy, or central cloudy dystrophy of Francois.

Question 120 of 130

When treating Acanthamoeba keratitis, what clinical course may be anticipated during treatment?

Successful topical therapies are not associated with significant ocular surface toxicity.

The infection cannot recur.

Rapidly successful treatment typically requires a short course of topical antibiotics.

Long-term therapy, including both topical and systemic agents, may be necessary.

Please select an answer

Feedback: Acanthamoeba keratitis can be difficult to diagnose and treat. Treatment with topical and sometimes oral

medications may be indicated, and the topical medications are quite toxic to the ocular surface. Even when the infection is

diagnosed early, treatment lasts 3-4 months. In cases of late diagnosis, treatment of up to 1 year is common. Recurrences

of the infection are a known complication.

Page 57: Cornea/External Disease

Question 121 of 130 The deposits seen in Schnyder crystalline corneal dystrophy are composed of what substances?

Collagen type II and hyaluronic acid

Cholesterol and phospholipids

Amyloid

Glylcosaminoglycan and lipid

Please select an answer Feedback: Schnyder crystalline dystrophy is a rare, slowly progressive, autosomal-dominant stromal dystrophy. The deposits consist of unesterified and esterified cholesterol and phospholipids. Glycosaminoglycan and lipid characterizes the deposits in Fleck corneal dystrophy, and amyloid is deposited in lattice dystrophy. Collagen type II and hyaluronic acid are constituents of the vitreous.

Question 122 of 130 You wish to start oral dapsone therapy for a patient with ocular mucous membrane pemphigoid. For what potentially complicating disorder should you check for first?

Glucose-6-phosphate dehydrogenase (G6PD) deficiency

High blood pressure

Penicillin allergy

Sickle cell disease

Please select an answer Feedback: Dapsone must be used cautiously in patients with G6PD deficiency and in those with sulfa allergies, as these patients can develop hemolytic anemia. Although a penicillin allergy, elevated blood pressure, and sickle cell disease status are not specific risk factors for this complication. It is important to note, however, that even patients without G6PD may develop hemolytic anemia, although at a lesser frequency. Therefore, the clinician should remain alert to any symptoms of lethargy or anemia. A hemaglobin or hematocrit should be monitored at regular intervals in these patients.

Question 123 of 130

What chemical damage is characterized by by soponification of fatty acids in cell membranes, tissue pH elevation, and intense intraocular inflammation from penetration of the anterior chamber?

Organic solvent

Weak acid

Strong acid

Strong alkali

Please select an answer Feedback: The scenario best describes injury caused by strong alkalis. Weak alkalis can cause soponafication, but do not typically penetrate into the anterior chamber. Acids, weak or strong, do not cause soponification.

Page 58: Cornea/External Disease

Question 124 of 130

What aspect of fibrin glue has the most potential concern for disease transmission?

Contains porcine blood products

Contains human serum

Can act as a bacterial growth media

Contains pooled human plasma and bovine products

Please select an answer Feedback: Recently, the use of fibrin glue has gained widespread popularity in various types of ocular surface surgeries. The use of this adhesive reduces the need for sutures. These uses are all off-label from the FDA-approved indication, and the potential for disease transmission exists because the adhesive contains both pooled human plasma and bovine products. Fibrin coagulum may act as a bacterial substrate, but behaves no differently than endogenous fibrin matrix. No porcine proteins are present in fibrin formulations. Fibrin contains purified protiens, not whole human serum.

Question 125 of 130

For a patient with adequate capsular support, what intraocular lens (IOL) would be preferred for placement in the ciliary sulcus?

One-piece acrylic posterior-chamber IOL

Plate silicone IOL

Acrylic posterior-chamber IOL with prolene haptics

Anterior-chamber IOL

Please select an answer Feedback:

An acrylic posterior-chamber IOL with prolene haptics would be the preferred lens for ciliary sulcus placement. Plate silicone lenses and one-piece acrylic posterior-chamber IOLs are designed only for in-the-bag placement. Anterior-chamber IOLs have no role in the posterior segment.

Question 126 of 130 What is the most common pathogen(s) responsible for infectious keratitis in contact lens wearers?

Streptococcal species

Staphylococcal species

Pseudomonas aeruginosa

Fungi

Please select an answer Feedback: Overall, the most common organisms causing infectious keratitis in North America include Staphylococcal species and streptococcal species, along with Pseudomonas aeruginosa. However, among contact lens wearers, P. aeruginosa is the most common bacterial cause.

Page 59: Cornea/External Disease

Question 127 of 130

Which topical medication is used as a prophylaxic agent for allergic conjunctivitis?

Prednisolone

Ketorolac

Phenylephrine

Cromolyn sodium

Please select an answer Feedback: Topical mast cell stabilizing agents such as cromolyn sodium are useful as prophylactic therapy for seasonal allergic conjunctivitis. Because their effects usually require continued use for more than a week, they are generally ineffective in the acute phase of of the disorder.

Question 128 of 130 In infectious keratitis, which adverse process is exacerbated by enzymes released by neutrophils and matrix metalloproteinases activated within the cornea?

Bacterial adherence

Neovascularization

Necrosis

Apoptosis

Please select an answer Feedback: In infectious keratitis, the enymes that are released by neutrophils and the activated within the corneal matrix exacerbate inflammatory necrosis. Apoptosis does not generally occur in this setting. Neovascularization occurs during the wound healing phase. These enzymes would inhibit bacterial adherence.

Question 129 of 130

For what type of burn does the buffering capacity of the ocular surface tissues mitigate harmful effects?

Ultraviolet

Thermal

Acid

Alkali

Please select an answer Feedback:

Acids cause denaturation and precipitation of proteins, which neutralize or buffer the reaction. Therefore acid solutions tend to cause less damage to ocular surface tissues than alkali solutions which are not buffered by the tissues. Thermal and radiation (ultraviolet) burns are unaffected by the chemical buffers in the tissues.

Page 60: Cornea/External Disease

Question 130 of 130

What variant of lattice corneal dystrophy is characterized by a typical facial appearance, blepharochalasis, pendulous ears, cranial and peripheral nerve palsies, and dry lax skin?

Type I (Biber-Haab-Dimmer)

Type III

Type II (Meretoja syndrome)

Type IV

Please select an answer

Feedback: In addition to lattice corneal dystrophy, there is systemic amyloid deposition in Meretoja syndrome resulting in

a characteristic facial mask, blepharochalasis, pendulous ears, cranial and peripheral nerve palsies, and dry lax skin. The

lattice lines in the cornea are less numerous and more peripheral than in Biber-Haab-Dimmer, which is the classic form of

lattice dystrophy. In Type III lattice, the deposits are midstromal and larger than in Type I; and in Type IV, the deposits are

large, nodular, and in the deep stroma.

Collected from www.aao.org (self assessment)

By Dr. AlBaraa AlQassimi